PIB January Merged

You might also like

Download as pdf or txt
Download as pdf or txt
You are on page 1of 354

PIB

24*7
1st – 3rd January 2024
www.anujjindal.in

Q1: The Ministry of Heavy Industries recently has extended the tenure of Production
Linked Incentive (PLI) Scheme for Automobile and Auto Components by 1 year and made
partial amendments to it. This PLI scheme was launched in ___________ with a
budgetary outlay of ___________. Fill the gaps
[a] 2022, Rs. 18,750 crores
[b] 2021, Rs. 25,938 crores
[c] 2023, Rs. 23,893 crores
[d] 2020, Rs. 27,334 crores
[e] 2019, Rs. 29,987 crores

Solution: [b]
www.anujjindal.in

PLI Scheme for


Automobiles and Auto
Components
The Ministry of Heavy Industries has made
following announcements with respect to
Production Linked Incentive (PLI) Scheme for
Automobile and Auto Components
• Extended the tenure of the Scheme by 1
year.
• Made partial amendments to scheme
www.anujjindal.in

About PLI Scheme for Automobiles and Auto


Components
• Objectives:
▪ To boost domestic manufacturing of advanced automotive
technology (AAT) products and attract investments in the
automotive manufacturing value chain.
▪ To overcome cost disabilities, create economies of scale, and
build robust supply chain in areas of advanced automotive
technology products.

• Launch Year: 2021

• Budgetary outlay: Rs. 25,938 crores.


www.anujjindal.in

About PLI Scheme for Automobiles and Auto


Components

• Beneficiaries: Both existing automotive companies as well as


new investors.

• Two parts of Scheme:


▪ Champion OEM: For electric or hydrogen-powered vehicles.
▪ Component Champions: For high-value and high-tech
components.
www.anujjindal.in

About PLI Scheme for Automobiles and Auto


Components
Recent Amendments
• Incentive will be applicable for 5 consecutive financial years,
starting from the FY 2023-24.
• Disbursement of the incentive will take place in the following FY
2024-25.
• Approved applicant will be eligible for benefits for 5 consecutive
financial years, but not beyond the FY ending on March 31,
2028.
• If an approved company fails to meet the threshold for an
increase in sales in first year, it will not receive any incentive for
that year.
www.anujjindal.in

Q2: Which program has been launched by Ministry of AYUSH to promote robust clinical
studies in priority areas of Ayurveda with Ayurveda academic institutions/hospitals
across the country through mutual collaboration?
[a] SMART 2.0
[b] AYUSHAM
[c] TRADMED
[d] PROAYUSH
[e] AYUSH AAROGYA

Solution: [a]
www.anujjindal.in

SMART 2.0 Program


Ministry of AYUSH has launched 'SMART
2.0' (Scope for Mainstreaming Ayurveda
Research among Teaching
professionals) Program to promote
robust clinical studies in priority areas
of Ayurveda with support of Ayurveda
academic institutions/hospitals
www.anujjindal.in

About SMART 2.0


• Launched By: Central Council for Research in Ayurvedic Sciences
(CCRAS) along with National Commission for Indian System of
Medicine.

• It will focus on safety, tolerability and adherence to Ayurveda


formulations in the priority research areas of Malnutrition,
Insufficient lactation, Abnormal Uterine Bleeding, Osteoporosis
in post-menopausal women and Diabetes Mellitus II.

• SMART 1.0: Under it, around 10 diseases were covered with the
active participation of teaching professionals from 38 colleges.
www.anujjindal.in

Q3: Ministry of Health and Family Welfare has recently achieved milestone of screening
more than 1 crore people under the National Sickle Cell Anaemia Elimination Mission.
This mission targets screening approximately __________ people over a period of
_________ years. Fill the gaps
[a] 4 crore, 4 years
[b] 5 crore, 5 years
[c] 7 crore, 3 years
[d] 6 crore, 2 years
[e] 3 crore, 6 years

Solution: [c]
www.anujjindal.in

National Sickle Cell


Anaemia
Elimination Mission
Ministry of Health and Family Welfare
has achieved milestone of screening
more than 1 crore people under the
National Sickle Cell Anaemia Elimination
Mission.
www.anujjindal.in

National Sickle Cell Anaemia


Elimination Mission
• Objective: To eliminate sickle cell genetic transmission by the
year 2047.

• Launch Year: 2023 (from Shahdol, Madhya Pradesh).

• It is being implemented in a mission mode as part of the


National Health Mission (NHM) for screening, prevention, and
management of sickle cell anemia disease in high prevalent
States/UTs of India.
www.anujjindal.in

National Sickle Cell Anaemia


Elimination Mission
• Focus Districts/States: 278 districts with higher prevalence of
SCD in 17 states viz., Gujarat, Maharashtra, Rajasthan, Madhya
Pradesh, Jharkhand, Chhattisgarh, West Bengal, Odisha, Tamil
Nadu, Telangana, Andhra Pradesh, Karnataka, Assam, Uttar
Pradesh, Kerala, Bihar and Uttarakhand.

• Screening Target: 7.0 crore people over a period of three years,


spanning from the fiscal year 2023-24 to 2025-26
www.anujjindal.in

Q4: With which organisation has NHAI signed an MoU to develop "Green Cover Index"
for the extensive network of National Highways in India?
[a] TERI
[b] IIT New Delhi
[c] IISc Bengaluru
[d] NITI Aayog
[e] NRSC

Solution: [e]
www.anujjindal.in

Green Cover Index:


NHAI-NRSC MoU
National Highways Authority of India has
entered into an MoU with the ISRO’s
National Remote Sensing Centre to
develop and report a "Green Cover
Index" for the extensive network of
National Highways in India
www.anujjindal.in

About MoU

• Under it, NRSC will undertake a comprehensive pan-India


estimation of green cover, called as "Green Cover Index" for
National Highways using high-resolution satellite imagery.

• Duration: 3 Years

• Background
▪ As per the Green Highways Policy 2015, Ministry of Road
Transport and Highways and NHAI has prioritize greening
highway corridors.
www.anujjindal.in

Q5: Where has India’s first all-girls ‘Samvid Gurukulam Girls Sainik School’ been
inaugurated recently?
[a] Vrindavan
[b] Jaipur
[c] Ludhiana
[d] Pune
[e] Patna

Solution: [a]
www.anujjindal.in

India’s First All-Girls


Sainik School
Union Defence Minister Rajanath
Singhhas has inaugurated India’s first
all-girls sainik school – ‘Samvid
Gurukulam Girls Sainik School’ in
Vrindavan, Uttar Pradesh.
www.anujjindal.in

About India’s 1st All-Indian Girls Sainik School

• Objective: To provide quality education, better career


opportunities, and encourage girls to join the Armed Forces.
• It will offer a CBSE curriculum along with military training for its
students.

• It will have capacity for 120 seats and admission will be done
through a written test.

• It will play important role in furthering the Government's


initiative to establish 100 new Sainik Schools across all
States/UTs, aligned with the objectives of the National
Education Policy 2020.
www.anujjindal.in

Q6: Kochi-Lakshadweep islands submarine optical fibre connection (KLI-SOFC) project


has been inaugurated recently to provide submarine cable internet connectivity from
Mainland (Kochi) to eleven Lakshadweep Islands. What is total link distance of this
project?
[a] 1,868 kilometres
[b] 1,540 kilometres
[c] 1,356 kilometres
[d] 1,267 kilometres
[e] 1,097 kilometres

Solution: [a]
www.anujjindal.in

KLI-SOFC project
Prime Minister Narendra Modi has
inaugurated Kochi-Lakshadweep islands
submarine optical fiber connection (KLI-
SOFC) project to provide submarine
cable internet connectivity from
Mainland (Kochi) to eleven
Lakshadweep Islands.
www.anujjindal.in

About KLI-SOFC Project


• Total link distance: 1,868 kilometres.

• Total cost of project: Rs.1072 crore plus taxes.


▪ It is funded by Universal Services Obligation Fund (USOF),
Department of Telecommunications

• Project Executing Agency: Bharat Sanchar Nigam Limited (BSNL)

• It will enable faster and more reliable internet services,


telemedicine, e-governance, educational initiatives, digital
banking, digital currency usage, digital literacy, etc
www.anujjindal.in
www.anujjindal.in

Q7: India is the _______________ culture shrimp producer in the World. Shrimp
contributes to more than ___________ of the total seafood export of India in value
terms. Fill the gaps
[a] 5th, 40%
[b] 4th, 45%
[c] 3rd, 50%
[d] 2nd, 55%
[e] 1st, 65%

Solution: [e]
www.anujjindal.in

Q8: Which state has recently created Guinness World Record for the most people
performing Surya Namaskar simultaneously at 108 venues?
[a] Uttarakhand
[b] Maharashtra
[c] Gujarat
[d] Uttar Pradesh
[e] Bihar

Solution: [c]
www.anujjindal.in

Q9: India is now the world’s …….. largest energy consumer, …………. largest consumer of
oil, ……………. Largest LPG consumer, ………….. largest LNG importer, ……………. largest
refiner and …………… largest automobile market in the world. Fill the gaps

[a] 3rd, 4th, 2nd, 4th, 5th, 6th


[b] 4th, 4th, 4th, 3rd, 3rd, 3rd
[c] 3rd, 3rd, 3rd, 4th, 4th, 4th
[d] 2nd, 3rd, 1st, 3rd, 5th,3rd
[e] 1st, 2nd, 3rd, 4th, 5th, 6th

Solution: [c]
www.anujjindal.in

Q10: Which States/UTs are being covered under the


Sagar Parikrama (Phase X) organised by Ministry of
Fisheries, Animal Husbandry and Dairying? [a] 1, 2 and 3 Only
(1) West Bengal [b] 2, 3 and 5 Only
(2) Puducherry [c] 1 and 3 Only
(3) Odisha [d] 2 and 4 Only
(4) Andhra Pradesh [e] 4 and 5 Only
(5) Tamil Nadu

Solution: [d]
www.anujjindal.in
PIB
24*7
4th – 5th January 2024
www.anujjindal.in

Q1: Identify incorrect statement about the recently launched “Prerana: An Experiential
Learning program”.
[a] It has been launched by the Ministry of Education
[b] It aims to offer a meaningful, unique, and inspiring experience to all participants,
thereby empowering them with leadership qualities
[c] It is a week-long residential program for 20 selected students (10 boys and 10 girls) of
class IX-XII for each batch
[d] It will run from a Vernacular School, established in 1888, in Vadnagar (Gujarat)
[e] Its curriculum has been prepared by IIT Bombay based on nine value-based themes

Solution: [e]
www.anujjindal.in

PRERANA Program
Ministry of Education launched the
“Prerana: An Experiential Learning
program” to empower participants with
leadership qualities
www.anujjindal.in

About PRERANA Program

• Objective: To offer a meaningful, unique, and inspiring


experience to all participants, thereby empowering them with
leadership qualities.

• Targeted Group: It is a week-long residential program for


selected students of class IX-XII.
▪ 20 Students are selected for each batch (10 boys and 10
girls)
www.anujjindal.in

About PRERANA Program


• Implementing Organisations:
▪ It will run from a Vernacular School, established in 1888, in
Vadnagar (Gujarat)
▪ Its curriculum has been prepared by IIT Gandhi Nagar based
on nine value-based themes.

• It will integrate principles of the Indian education system and


the philosophy of value-based education which is a cornerstone
of the National Education Policy (NEP) 2020.
www.anujjindal.in

Q2: The Union Cabinet has recently approved the overarching scheme “PRITHvi VIgyan
(PRITHVI)” of the Ministry of Earth Sciences, for implementation during the period of
___________ at an overall cost of ___________. Fill the gaps
[a] 2021-26, Rs. 4,797 crores
[b] 2022-27, Rs. 5,356 crores
[c] 2023-28, Rs. 6,560 crores
[d] 2024-29, Rs. 7,204 crores
[e] 2025-30, Rs. 8,856 crores

Solution: [a]
www.anujjindal.in

PRITHvi VIgyan (PRITHVI)


Scheme
The Union Cabinet has approved the
overarching scheme “PRITHvi VIgyan
(PRITHVI)” of Ministry of Earth Sciences
www.anujjindal.in

About PRITHVI Scheme


• Objectives
▪ To augment and sustain long-term observations of the
atmosphere, ocean, geosphere, cryosphere, and solid earth
to record the vital signs of the Earth System and change
▪ To develop modeling systems for understanding and
predicting weather, ocean, and climate hazards and
understanding the science of climate change
▪ To explore polar and high seas regions of the Earth towards
the discovery of new phenomena and resources
▪ To develop technology for exploration and sustainable
harnessing of oceanic resources for societal applications
www.anujjindal.in

About PRITHVI Scheme


• Duration: 2021-26

• Overall cost: Rs. 4,797 crores

• Sub-Schemes/Components: It encompasses five ongoing sub-


schemes namely
▪ Atmosphere & Climate Research-Modelling Observing
Systems & Services (ACROSS)
▪ Ocean Services, Modelling Application, Resources and
Technology (O-SMART)
▪ Polar Science and Cryosphere Research (PACER)
▪ Seismology and Geosciences (SAGE)
▪ Research, Education, Training and Outreach (REACHOUT).
www.anujjindal.in

Q3: The Ministry of Environment, Forests and Climate Change for the first time has
submitted three nominations from India for Wetland City Accreditation (WCA) under the
Ramsar Convention on Wetlands. Which of the following are nominated cities for WCA
from India?
(1) Indore [a] 1, 2 and 3 Only
(2) Jaipur [b] 1, 3 and 5 Only
(3) Bhopal [c] 2, 3 and 4 Only
(4) Varanasi [d] 3, 4 and 5 Only
(5) Udaipur [e] 2, 3 and 5 Only

Solution: [b]
www.anujjindal.in

Wetland City Accreditation


Ministry of Environment, Forests and
Climate Change has submitted three
nominations from India for Wetland City
Accreditation (WCA) under the Ramsar
Convention on Wetlands.
www.anujjindal.in

Three Nominated Cities

▪ Indore (Madhya Pradesh)


▪ Bhopal (Madhya Pradesh)
▪ Udaipur (Rajasthan)

• These are the first three Indian cities for which nominations
have been submitted for WCA
www.anujjindal.in

About Wetland City Accreditation


• Objective: To promote the conservation and wise use of
wetlands, as well as sustainable socio-economic benefits for
local populations.

• It is a voluntary accreditation system to recognize the


importance of wetlands in urban and peri-urban environments.

• It was approved in the COP12 of the Ramsar Convention (2015).

• Criteria: Its accreditation is based on 6 criteria which include the


presence of Ramsar sites in the city, or other wetland
conservation sites etc.
▪ Once granted it is valid for 6 years.
www.anujjindal.in

Q4: With which country has India signed an MoU for supporting Indian Railways to
achieve Mission Net Zero Carbon Emission by 2030 with technical assistance from the
South Asia Regional Energy Partnership (SAREP)?
[a] Germany
[b] France
[c] USA
[d] Australia
[e] Japan

Solution: [c]
www.anujjindal.in

India-US MoU to support


Indian Railways
Union Cabinet has approved an MoU
between India and the United States for
International Development/India
(USAID/India). It will support Indian
Railways to achieve Mission Net Zero
Carbon Emission by 2030.
www.anujjindal.in

About MoU
• It will provide a platform for Indian Railways to interact and
share the latest developments and knowledge in the railway
sector.

• Duration: 5 years or until the effective end of the South Asia


Regional Energy Partnership (SAREP) whichever period is
shorter.

• Technical assistance for MoU: It will be provided by USAID under


the SAREP initiative.
www.anujjindal.in

About SAREP
• Objective: To improve access to affordable, secure, reliable, and
sustainable energy across six countries (Bangladesh, Bhutan,
India, Maldives, Nepal, and Sri Lanka).

• It is the flagship regional energy program of USAID/India.

• Tenure: 2021-26
www.anujjindal.in

Q5: Since its launch in 2019, Jal Jeevan Mission (JJM) has crossed the milestone of
providing tap water connections to 14 Crore (72.71%) rural households. Which of the
following States/UTs so far have achieved 100% coverage under JJM?
(1) Gujarat
(2) Maharashtra [a] 2, 4 and 5 Only
(3) Haryana [b] 1, 2 and 3 Only
(4) Puducherry [c] 2, 3 and 4 Only
(5) Uttar Pradesh [d] 1, 3 and 4 Only
[e] 3, 4 and 5 Only

Solution: [d]
www.anujjindal.in

Jal Jeevan Mission


Milestone
Jal Jeevan Mission (JJM) has crossed
milestone of providing tap water
connections to 14 Crore (72.71%) rural
households since its launch in 2019.
www.anujjindal.in

Several milestones of JMM


• Six states and three UTs have achieved 100% coverage:
▪ States: Goa, Telangana, Haryana, Gujarat, Punjab, and
Himachal Pradesh
▪ UTs: Puducherry, D&D and D&NH, and A&N Islands

• More than 2 lakh villages and 161 districts have been now
declared ‘Har Ghar Jal.’

• It has ensured tap water supply in 9.24 lakh (90.65%) schools


and 9.57 lakh (86.63%) Anganwadi centres nationwide.

• In the 112 aspirational districts, tap water access has surged


from 21.41 lakh (7.86%) households in 2019 to 1.96 Crore
(72.08%) households.
www.anujjindal.in
www.anujjindal.in

Q6: The ‘e-samridhi portal’ was launched for registration, purchase and payment of
____________ producing farmers. Fill the gap
[a] Arhar dal
[b] Chickpea dal
[c] Channa dal
[d] Moong dal
[e] Tur dal

Solution: [e]
www.anujjindal.in

Q7: India, globally ranks __________ in Renewable Energy Installed Capacity, _________
in Wind Power capacity, and _________ in Solar Power capacity. Fill the gaps
[a] 1st, 2nd and 3rd
[b] 4th, 4th and 5th
[c] 3rd, 4th and 6th
[d] 2nd, 4th and 1st
[e] 1st, 3rd and 4th

Solution: [b]
www.anujjindal.in

Q8: With which state Government has Power Finance Corporation Ltd. (PFC) recently
signed an MoU to provide comprehensive financial backing for the state's generation,
transmission, and distribution projects?
[a] Karnataka
[b] Maharashtra
[c] Gujarat
[d] Andhra Pradesh
[e] Tamil Nadu

Solution: [c]
www.anujjindal.in

Q9: With which organisation has Khadi and Village Industries Commission recently
signed MoU to enhance the quality of Khadi products, empower artisans and offer
quality products under the 'Made in India' banner for Khadi?
[a] QCI
[b] NITI Aayog
[c] IIT Bombay
[d] EXIM Bank
[e] ITPO

Solution: [a]
www.anujjindal.in

Q10: REC Limited has entered into an MoU with Rail Vikas Nigam Limited (RVNL) to
finance up to _____________ for Infrastructure Projects to be executed by RVNL over the
next _____________. Fill the gaps
[a] Rs. 45,000 Crores, 15 years
[b] Rs. 40,000 Crores, 10 years
[c] Rs. 35,000 Crores, 5 years
[d] Rs. 30,000 Crores, 3 years
[e] Rs. 25,000 Crores, 7 years

Solution: [c]
www.anujjindal.in
PIB
24*7
6th – 8th January 2024
www.anujjindal.in

Q1: Project Veer Gatha has been launched in 2021 under Gallantry Awards Portal in 2021
to provide platform to the school students to do creative projects/activities based on
gallantry award winners. It is a joint initiative of which Ministry/ies?
[a] Ministry of Home Affairs
[b] Ministry of Defence
[c] Ministry of Education
[d] A and B
[e] B and C

Solution: [e]
www.anujjindal.in

Project Veer Gatha 3.0


The third edition of Project ‘Veer Gatha’
has witnessed participation of around
1.37 crore students from around 2.43
lakh schools, with 100 winners (Super
100) being selected at the national level.
www.anujjindal.in

About Project Veer Gatha


• Objectives:
▪ To disseminate the details of acts of bravery of the Gallantry
awardees and their life stories among the students
▪ To raise the spirit of patriotism and instil values of civic
consciousness amongst them.

• It is a joint initiative of Ministry of Defence and Ministry of


Education as part of Republic Day celebrations.
www.anujjindal.in

About Project Veer Gatha


• It was launched in 2021 as part of ‘Azadi Ka Amrit Mahotsav’ to
celebrate 75th year of Independence.

▪ It has been instituted under Gallantry Awards Portal to


provide platform to the school students to do creative
projects/activities based on gallantry award winners.
www.anujjindal.in

About Project Veer Gatha 3.0


• It was launched in July 2023.

• 100 winners (super 100) under it includes 25 students each from


Class 3rd to 5th; Class 6th to 8th; Class 9th to 10th & Class 11th
to 12th categories.

• Each winner is awarded a cash prize of Rs.10,000 and their


felicitation is done jointly by the Ministry of Education &
Ministry of Defence in New Delhi.
www.anujjindal.in

Q2: Which Central Sector Scheme has won Best Innovation Award for Innovation
Sandbox presentation during Public Policy Dialogues–2024 at Indian School of Business,
Hyderabad?
[a] MNREGA Scheme
[b] Krishi Sinchayee Yojana
[c] Pradhan Mantri Awas Yojana
[d] SVAMITVA Scheme
[e] PM SVANidhi Scheme

Solution: [d]
www.anujjindal.in

SVAMITVA Scheme
SVAMITVA Scheme of Ministry of
Panchayati Raj has won Best Innovation
Award for Innovation Sandbox
presentation during Public Policy
Dialogues–2024 held at Indian School of
Business, Hyderabad
www.anujjindal.in

About SVAMITVA Scheme


• Objective: To provide the ‘Record of Rights’ to village household
owners possessing houses in inhabited areas (Abadi) in village

• Launch Year: 2021

• Nodal Ministry: Ministry of Panchayati Raj

• Type: Central Sector Scheme (100% funded by the Centre)

• Target year of completion of its implementation: 2024–25


www.anujjindal.in

About SVAMITVA Scheme


• Features:

▪ Under it, land parcels in rural inhabited area of all the


villages are surveyed using cutting-edge drone surveys and
GIS mapping technology.

▪ It ensures accurate demarcation of land parcels and


providing individuals with property records of rights i.e.
SVAMITVA Property Cards for determination of clear
ownership of property.
www.anujjindal.in

About SVAMITVA Scheme


• Features:

▪ It facilitates monetisation of properties leading to ease of


securing bank loans, reduction of property related disputes,
comprehensive village level planning

▪ It also provides a basis for assessment of property tax, which


would accrue to the Gram Panchayats directly in States
where it is devolved.
www.anujjindal.in

About SVAMITVA Scheme


• Four identified broad areas of Impact of SVAMITVA Scheme are
www.anujjindal.in

About SVAMITVA Scheme


• Implementation: It is being implemented with the collaborative
efforts of the Ministry of Panchayati Raj, Survey of India (SoI),
State Revenue Department, State Panchayati Raj Department
and National Informatics Centre.

▪ States need to sign Memorandum of Understanding (MoU)


with SoI for implementation of the scheme.

▪ So far 31 States/UTs have signed MoU with SoI.


www.anujjindal.in

About SVAMITVA Scheme


• Other Awards won by SVAMITVA Scheme
▪ Gold Prize in National Awards for e-Governance 2023: It had
won in the category “Use of Emerging Technology for
Providing Citizen-Centric Services”.
✓ It was awarded by Ministry of Personnel, Public
Grievances and Pensions at Indore, Madhya Pradesh in
October 2023.
▪ Gold award for the category “Innovative Use of Technology
in e-Governance for Digital Transformation” at Digitech
Conclave 2023 organized at Goa in August 2023.
www.anujjindal.in

Q3: Which State/UT will be covered under the Sagar Parikrama Phase- XI of Ministry of
Fisheries, Animal Husbandry & Dairying?
[a] Odisha
[b] West Bengal
[c] Assam
[d] Andaman & Nicobar
[e] Lakshadweep

Solution: [a]
www.anujjindal.in

Sagar Parikrama Phase- XI


Ministry of Fisheries, Animal Husbandry
& Dairying is organising Sagar
Parikrama Phase- XI event in Odisha.
www.anujjindal.in

About Sagar Parikrama Phase- XI


• It will cover the coastal districts of Odisha namely Ganjam, Puri,
Jagatsinghpur, Kendrapara, Bhadrak, Balasore district.

• It will disseminate best practices and initiatives taken through


Pradhan Mantri Matsya Sampada Yojana (PMMSY) scheme, KCC,
and other schemes to fishermen.
www.anujjindal.in

About Sagar Parikrama


• Objectives:
▪ To facilitate interaction with fishermen, coastal
communities, and stakeholders
▪ To disseminate information on various fisheries related
schemes and programs being implemented

• Organising Ministry: Ministry of Fisheries, Animal Husbandry &


Dairying
• 1st phase of “Sagar Parikrama” was started from Mandvi,
Gujarat in March 2022.
▪ So far, the total 10 phases of Sagar Parikrama have been
launched.
www.anujjindal.in

Q4: When was iDEX (Innovations for Defence Excellence) launched by Ministry of
Defence to cultivate an innovation ecosystem in the Defence and Aerospace sector by
collaborating with startups, innovators, MSMEs, incubators, and academia?
[a] 2023
[b] 2016
[c] 2022
[d] 2020
[e] 2018

Solution: [e]
www.anujjindal.in

IDEX- DIO
Innovations for Defence Excellence-
Defence Innovation Organization (iDEX-
DIO) will participate in the tenth edition
of the Vibrant Gujarat Summit 2024 at
Gandhinagar, Gujarat.
www.anujjindal.in

About iDEX (Innovations for


Defence Excellence)
• Objective: To cultivate an innovation ecosystem in the Defence
and Aerospace sector by collaborating with startups, innovators,
MSMEs, incubators, and academia.

• Nodal Ministry: Ministry of Defence

• Launch Year: 2018

• It offers grants and support for R&D with significant potential


for future adoption in Indian defence and aerospace.
• It has received the PM Award for Innovation in the defence
sector.
www.anujjindal.in

Q5: Where has been India’s first Healthy & Hygienic Food Street named “PRASADAM’
inaugurated recently?
[a] Varanasi
[b] Jaipur
[c] Ujjain
[d] New Delhi
[e] Bengaluru

Solution: [c]
www.anujjindal.in

PRASADAM
India’s first Healthy & Hygienic Food
Street “PRASADAM” has been recently
inaugurated at Neelkanth Van, Mahakal
Lok, in Ujjain, Madhya Pradesh.
www.anujjindal.in

About PRASADAM
• It will connect common citizens in every corner of the country
with pure and safe local and traditional food.

• It also aligns common people and tourists to safe and healthy


eating habit.

• It will offer convenient and culturally rich dining options for the
1-1.5 lakh devotees who visit the Mahakaleshwar Temple daily.

• It is designed to provide various facilities including a kids' play


area, drinking water facility, CCTV surveillance, parking, public
conveniences and seating spaces.
www.anujjindal.in

Other Initiatives launched


• Mannhit app: It has been launched by Madhya Pradesh
Government (National Health Mission) to facilitate screening for
mental health.

• The DART Book: It has been released by the Food Safety and
Standards Authority of India (FSSAI) to empower consumers to
tackle adulteration of common food items at home with simple
tests.

• Food Safety on Wheels (FSW): It is a mobile food testing van


launched to reach remote areas and conduct training and
awareness activities to promote adulteration testing.
www.anujjindal.in
www.anujjindal.in

Q6: With which bank has REC Limited recently signed an MoU to facilitate joint
sanctions of loans to fund power, infrastructure and logistics projects in the country over
the next three years?
[a] State Bank of India
[b] Bank of Baroda
[c] Indian Overseas Bank
[d] Canara Bank
[e] Bank of Maharashtra

Solution: [b]
www.anujjindal.in

Q7: The Union Cabinet has recently approved the proposal to name which airport as
“Maharishi Valmiki International Airport”?
[a] Jaipur
[b] Ujjain
[c] Ayodhya
[d] Varanasi
[e] Indore

Solution: [c]
www.anujjindal.in

Q8: Where is Bharat Mobility Global Expo 2024, the first of its kind global mobility show
covering the entire mobility ecosystem being organised by Industry associations with the
support from Government of India?
[a] Chennai
[b] Mumbai
[c] Hyderabad
[d] Dehradun
[e] New Delhi

Solution: [e]
www.anujjindal.in

Q9: With which country has the Union Cabinet recently approved signing of MoU on
cooperation in the hydrocarbon sector?
[a] Saudi Arabia
[b] Russia
[c] Niger
[d] Chad
[e] Guyana

Solution: [e]
www.anujjindal.in

Q10: Union Cabinet has recently approved signing of MoU between India and which
country for the development of a Joint Small Satellite?
[a] Maldives
[b] Bhutan
[c] Seychelles
[d] Mauritius
[e] Bangladesh

Solution: [d]
www.anujjindal.in

Q11: Coal India Limited under its CSR initiative has recently signed MoU with EdCIL
(India) Limited for digitizing education in eleven districts of which state?
[a] Jharkhand
[b] West Bengal
[c] Chhattisgarh
[d] Bihar
[e] Odisha

Solution: [a]
www.anujjindal.in

Q12: Which PSU has recently entered into MoU with National Skill Development
Corporation to set up Multi Skill Development Institutes under each subsidiary?
[a] ONGC
[b] BPCL
[c] CIL
[d] NHPC
[e] REC

Solution: [c]
www.anujjindal.in

Q13: With which industry body has Indian Railways signed an MoU to reduce energy and
water consumption lowering GHG emissions?
[a] CII
[b] FICCI
[c] ITPO
[d] ASSOCHAM
[e] FIEO

Solution: [a]
www.anujjindal.in

Q14: What was the theme of Ministry of MSME’s Pavilion at the Atmanirbhar Bharat
Utsav 2024 held in New Delhi to showcase and celebrate the self-reliance in India?
[a] MSME for Exports
[b] Growth through MSME
[c] MSME for India
[d] PM Vishwakarma
[e] Make in India

Solution: [d]
www.anujjindal.in

Q15: Which State/UT Government has launched Mannhit app under National Health
Mission to facilitate screening for mental health?
[a] Maharashtra
[b] Tamil Nadu
[c] New Delhi
[d] Madhya Pradesh
[e] Rajasthan

Solution: [d]
www.anujjindal.in
PIB
24*7
9th – 11th January 2024
www.anujjindal.in

Q1: Which of the following traditional AYUSH medicine systems have been recently
included in the International Classification of Diseases (ICD) 11 as Module 2 by the World
Health Organisation?
(1) Ayurveda [a] 2, 3 and 5
(2) Yoga [b] 3, 4 and 5
(3) Siddha [c] 1, 3 and 5
(4) Homoeopathy [d] 1, 2 and 3
(5) Unani [e] 2, 3 and 4

Solution: [c]
www.anujjindal.in

Traditional Medicine
Morbidity Codes in ICD 11
World Health Organization has launched
Traditional Medicine Morbidity Codes of
Ayurveda, Siddha, and Unani Chapter in
International Classification of Diseases
(ICD) 11 as Module 2
www.anujjindal.in

About ICDs
Significance of inclusion: It will lead to global uniformity in ASU
(Ayurveda, Unani, and Siddha) medicine as a code of vocabulary
defining diseases.

About ICDs
• WHO has developed a classification series called International
Classification of Diseases (ICDs) to classify diseases
internationally.
• It comprises the global data on diseases based on healthcare
practices to be diagnosed through modern biomedicine.
www.anujjindal.in

Q2: The Ministry of Railways under its “Startups for Railways” initiative has so far
awarded 23 projects valued at Rs. 43.87 Crores. When was this initiative launched?
[a] 2018
[b] 2020
[c] 2021
[d] 2022
[e] 2023

Solution: [d]
www.anujjindal.in

Startups for Railways


Initiative
Ministry of Railways under its “Startups
for Railways" initiative has so far
awarded 23 projects valued Rs. 43.87
Crores.
www.anujjindal.in

About Startups for Railways Initiative

• Objective: To leverage innovative technologies developed by


Indian Startups/MSMEs/Innovators/Entrepreneurs to improve
operational efficiency and safety of Indian Railways.

• Launch Year: 2022

• It seeks to address quality, reliability, and maintainability-


related issues of the Indian Railways.

• Under it, the beneficiary Startup/MSME/Innovator/


Entrepreneur will have exclusive ownership of Intellectual
Property Rights (IPR) created in the project.
www.anujjindal.in

Q3: Which bank has recently signed MoU with Deendayal Antyodaya Yojana - National
Rural Livelihoods Mission (Ministry of Rural Development) to facilitate and streamline
enterprise financing for rural Self-Help Group (SHG) women through its specialized
financial product Svyam Siddha?
[a] Bank of Baroda
[b] SBI
[c] ICICI Bank
[d] Canara Bank
[e] HDFC Bank

Solution: [b]
www.anujjindal.in

MoU - DAY-NRLM and SBI


DAY – NRLM and SBI have signed MoU
to facilitate and streamline enterprise
financing for rural Self-Help Group
(SHG) women through SBI’s specialised
financial product Svyam Siddha.
www.anujjindal.in

About Svyam Siddha


• It is a specialized financial product of SBI, exclusively tailored for
SHG women entrepreneurs seeking loans up to Rs 5 lakhs.

• It is designed to alleviate the challenges associated with


extensive documentation requirements for bank loan
applications and reduce the turnaround time
▪ It allows an easy application process where a simple loan
application with KYC details can be submitted at local SBI
bank branches.
• Under the MoU, DAY-NRLM will now facilitate the loan
application process and oversee loan repayment through its
dedicated field cadre.
www.anujjindal.in

Q4: Which organisation in collaboration with Deendayal Antyodaya Yojana - National


Rural Livelihoods Mission (Ministry of Rural Development) has launched Training Toolkit
to promote access to formal finance for SHG women entrepreneurs?
[a] NITI Aayog
[b] UNDP
[c] World Bank
[d] ADB
[e] WTO

Solution: [c]
www.anujjindal.in

Training Toolkit for SHG


women entrepreneurs
DAY – NRLM and World Bank have
launched Training Toolkit for SHG
women entrepreneurs to promote
access to formal finance for SHG women
entrepreneurs.
www.anujjindal.in

About Training Toolkit


• Objectives:
▪ To enhance the capacities of State Rural Livelihoods Missions
▪ To facilitate increased access to formal finance by SHG
members for their economic enterprises
• Developed in collaboration with the International Finance
Corporation under the World Bank-funded National Rural
Economic Transformation Project.
• It covers essential topics, including fundamentals of banking,
financial statements, and working capital calculations.
• Its key modules have been converted into animated videos to
enhance accessibility and facilitate self-capacity building for
entrepreneurs.
www.anujjindal.in

Q5: Which of the following states having Fifth Schedule Areas have yet not notified State
Rules concerning Strengthening of the Panchayats (Extension to Scheduled Areas) Act,
1996 (PESA Act, 1996) under their respective State Panchayati Raj Acts?
(1) Jharkhand
[a] 1, 2 and 3 Only
(2) Maharashtra
[b] 2 and 4 Only
(3) Madhya Pradesh
[c] 2, 3 and 5 Only
(4) Rajasthan
[d] 3 and 4 Only
(5) Odisha
[e] 1 and 5 Only

Solution: [e]
www.anujjindal.in

Regional Conference on
Strengthening of PESA
What: Ministry of Panchayati Raj
recently organised the Regional
Conference on Strengthening of the
Panchayats (Extension to Scheduled
Areas) Act, 1996 in Pune.
Why: To serve as a platform for sharing
insights, experiences, and way forward
in the effective implementation of the
PESA Act.
www.anujjindal.in

About Conference

• It is the first of the two Regional Conferences designed and


conceptualized by the Ministry of Panchayati Raj for PESA States

• It saw the participation of five States viz. Maharashtra, Madhya


Pradesh, Gujarat, Rajasthan, and Himachal Pradesh.
www.anujjindal.in

About PESA Act, 1996


• It was enacted by Parliament as per Article 243M(4)(b) to extend
Part IX of the Constitution, relating to Panchayats, to the 5th
Schedule Areas.
• It empowers legislatures of States, having 5th Schedule Areas, to
frame all laws concerning the extension of the provisions of Part
IX of the Constitution relating to the Panchayats in these Areas.

▪ 5th Scheduled Areas are those as referred to in clause (1) of


article 244 of the Constitution.
▪ Presently, 10 States have Fifth Schedule Areas viz. Andhra
Pradesh, Chhattisgarh, Gujarat, Himachal Pradesh,
Jharkhand, Madhya Pradesh, Maharashtra, Odisha,
Rajasthan and Telangana,
www.anujjindal.in

About PESA Act, 1996


• Out of the 10 PESA States, 8 States have framed and notified
their State PESA Rules under their respective State Panchayati
Raj Acts. They are
▪ Andhra Pradesh, Chhattisgarh, Gujarat, Himachal Pradesh,
Maharashtra, Madhya Pradesh, Rajasthan, and Telangana

• Two States (Jharkhand & Odisha) have also framed their draft
PESA Rules but have not yet notified these Rules.
www.anujjindal.in
www.anujjindal.in

Q6: Which Ministry has constituted a Committee for “Prevention and Regulation of
Misleading Advertisement in Coaching Sector” under the chairmanship of Rohit Kumar
Singh?
[a] Ministry of Education
[b] Ministry of Consumer Affairs, Food & Public Distribution
[c] Ministry of Commerce and Industry
[d] Ministry of Information and Broadcasting
[e] Ministry of Personnel, Public Grievances & Pensions

Solution: [b]
www.anujjindal.in

Q7: Ministry of Panchayati Raj in collaboration with which organization has recently
conducted the National Workshop on the Role of Panchayati Raj Institutions in
Addressing Gender-Based Violence?
[a] UNESCO
[b] UNFPA
[c] UNDP
[d] UNHCR
[e] UNFAO

Solution: [b]
www.anujjindal.in

Q8: Which program is being conducted by the Ministry of Social Justice and
Empowerment for creating platforms for Divyangjan to contribute to the socio-cultural
and economic growth of the nation?
[a] Divyangjan Protshan Manthan
[b] Divya Samridhi Utsav
[c] Divya Kala Shakti
[d] Divyangjan Samarthan
[e] Divyangjan Shakti

Solution: [c]
www.anujjindal.in

Q9: What is the theme of the 10th edition of the Vibrant Gujarat Global Summit being
held in Gandhinagar, Gujarat?
[a] ‘Gujarat Ready for Industry 4.0’
[b] ‘Make in Gujarat for World’
[c] 'Gateway to the Future'
[d] 'Gujarat Going Global'
[e] ‘Gujarat for India and World’

Solution: [c]
www.anujjindal.in

Q10: For modernization and upgradation of which fishing harbour has the Ministry of
Fisheries, Animal Husbandry & Dairying recently approved Rs 108.91 Crore with 100%
central financial assistance under the Pradhan Mantri Matsya Sampada Yojana?
[a] Paradip Fishing Harbour
[b] Kochi Fishing Harbour
[c] Chennai Fishing Harbour
[d] Karwar Fishing Harbour
[e] Sindhudurg Fishing Harbour

Solution: [a]
www.anujjindal.in
PIB
24*7
12th – 13th January 2024
www.anujjindal.in

Q1: Who amongst the following is/are not eligible to avail scholarship under the Ministry
of Education’s National Means-cum-Merit Scholarship Scheme?
[a] Student whose parental income from all sources is Rs. 5,50,000/- per annum
[b] Student having 45% marks or equivalent grade in Class VII examination
[c] Students of NVS, KVS, and residential schools
[d] A and C
[e] A, B and C

Solution: [e]
www.anujjindal.in

National Means cum Merit


Scholarship Scheme
Ministry of Education has extended the
last date for submission of applications
for National Means cum Merit
Scholarship Scheme for Year 2023-24 on
the National Scholarship Portal to 31st
January, 2024.
www.anujjindal.in

About National Means cum Merit


Scholarship Scheme
• Objective: To award scholarships to meritorious students of
economically weaker sections to arrest their dropout at class VIII
and encourage them to continue their education at secondary
stage.

• Launch Year: 2008


▪ It has been continued from FY 2021-22 to FY 2025-26 with
total allocation of Rs.1827 crores.

• Type: Central Sector Scheme.

• Amount of Scholarship: Rs. 12000/- per annum.


www.anujjindal.in

About National Means cum Merit


Scholarship Scheme
• Eligibility criteria for students
▪ Parental income should not be more than Rs. 3,50,000/- per
annum.

▪ Must have a minimum of 55% marks or equivalent grade in


the Class VII examination for appearing in the selection test
✓ 5% rebate for SC/ST students

▪ Must be studying as regular students in a Government,


Government-aided, and local body school.
▪ NOT Eligible: Students of NVS, KVS and residential schools.
www.anujjindal.in

Q2: How many pair/s with respect to winners of Swachh Survekshan awards 2023 are
incorrectly matched?
(1) All India Clean City Rank 1 – Surat Only [a] 1 Pair
(2) All India Clean City (Population < 1 Lakh) Rank 1 – Patan [b] 2 Pairs
(3) Cleanest Cantonment Board – Agra [c] 3 Pairs
(4) Best Safaimitra Surakshit Sheher – Chandigarh [d] 4 Pairs
(5) Best Performing State Rank 1 – Maharashtra [e] 5 Pairs

Solution: [c]
www.anujjindal.in

Swachh Survekshan Awards


2023
President of India Droupadi Murmu has
conferred Swachh Survekshan Awards
2023 at Bharat Mandapam, New Delhi
hosted by the Ministry of Housing and
Urban Affairs.
www.anujjindal.in

About Swachh Survekshan Awards 2023

• The winners were selected according to the Swachh Survekshan


2023, conducted by the MoHUA under the Swachh Bharat Urban
Mission 2.0.

• SS 2023 is the world’s largest urban sanitation and cleanliness


survey.

• Its theme was “Waste to Wealth”


Swachh Survekshan Awards 2023 Winners www.anujjindal.in
All India Clean City Rank 1 (Joint Winners): Indore
(topped for 6th consecutive year) and
Surat
Rank 3: Navi Mumbai
All India Clean City (Population < 1 lakh) Rank 1: Sasvad
Rank 2: Patan
Rank 3: Lonavala
Cleanest Cantonment Board Mhow
Best Safaimitra Surakshit Sheher Chandigarh

Cleanest Ganga Towns Rank 1: Varanasi


Rank 2: Prayagraj
Best Preforming State Rank 1: Maharashtra
Rank 2: Madhya Pradesh
Rank 3: Chhattisgarh
www.anujjindal.in

Q3: Where has Prime Minister Narendra Modi recently inaugurated India’s longest sea
bridge?
[a] Kolkata
[b] Kochi
[c] Chennai
[d] Surat
[e] Mumbai

Solution: [e]
www.anujjindal.in

Atal Bihari Vajpayee Sewri -


Nhava Sheva Atal Setu
What: Prime Minister Narendra Modi has
inaugurated Mumbai Transharbour link
(MTHL), now named ‘Atal Bihari
Vajpayee Sewri - Nhava Sheva Atal Setu’
Why: To improve ‘ease of mobility’ of
citizens by strengthening urban
transport infrastructure and
connectivity.
www.anujjindal.in

About Atal Bihari Vajpayee Sewri - Nhava


Sheva Atal Setu

• It is the longest bridge in India and also the longest sea bridge in
India.
• Total length: It is a 21.8 km long 6-lane bridge (16.5 km length
over the sea and 5.5 km on the land)
• Total Cost: Rs 17,840 crore.
• Location: Mumbai/Navi Mumbai
• It will provide faster connectivity to Mumbai International
Airport and Navi Mumbai International Airport
• It will also reduce the travel time from Mumbai to Pune, Goa,
and South India.
www.anujjindal.in

Q4: How much investment has been committed for the development of river cruise
tourism in the country by 2047 in the first meeting of the Inland Waterways
Development Council?
[a] Rs. 25,000 crores
[b] Rs. 30,000 Crores
[c] Rs. 35,000 crores
[d] Rs. 40,000 crores
[e] Rs. 45,000 crores

Solution: [e]
www.anujjindal.in

1st Inland Waterways


Development Council
Meeting
What: The first meeting of the Inland
Waterways Development Council (IWDC)
concluded in Kolkata, West Bengal.
Why: To strengthen the Inland Water
Transport system in the country.
www.anujjindal.in

About IWDC Meeting

• It was organized by the Inland Waterways Authority of India


(IWAI)

• It was chaired by the Union Minister of Ports, Shipping and


Waterways Sarbananda Sonowal

• Its primary focus is to leverage inland waterways as catalysts for


economic growth and commerce.
www.anujjindal.in

Key Outcomes of 1st IWDC Meeting


• Committed investment Rs.45,000 crore for the development of
river cruise tourism:
▪ Of this, Rs.35,000 crore has been earmarked for cruise
vessels and Rs. 10,000 crore for the development of cruise
terminal infrastructure at the end of Amrit Kaal by 2047.
• Harit Nauka- Guidelines for Green Transition of Inland Vessels
launched:
▪ It put forth a strong commitment towards furthering
passenger transport through waterways in an environment-
friendly and sustainable manner.
▪ This will be achieved by promoting the adoption of low-
emission fuel (CNG/LNG/electric/hydrogen/methanol) as
propulsion fuel for inland vessel operations (Green Vessels).
www.anujjindal.in

Key Outcomes of 1st IWDC Meeting


• River Cruise Tourism Roadmap, 2047 launched: It focuses on
four vital pillars, including
✓ Infrastructure
✓ Integration
✓ Accessibility
✓ Policy for promoting river cruise tourism

▪ As a part of it, over 30 possible routes and tourist circuits


along inland waterways have been identified for further
development.
www.anujjindal.in
www.anujjindal.in

Q5: Where has Prime Minister Narendra Modi recently inaugurated ‘Bharat Ratnam’
(Mega Common Facilitation Centre) for the Gems and Jewellery sector?
[a] Surat
[b] Mumbai
[c] Jaipur
[d] New Delhi
[e] Kolkata

Solution: [b]
www.anujjindal.in

Q6: Which State has launched Namo Mahila Shashaktikaran Abhiyaan to empower
women by providing skill development training and exposure to entrepreneurship
development?
[a] Karnataka
[b] Madhya Pradesh
[c] Rajasthan
[d] Chhattisgarh
[e] Maharashtra

Solution: [e]
www.anujjindal.in

Q7: Who has been awarded the “Indian of the Year Award" for the year 2023 in the
category ‘Outstanding Achievement’?
[a] DRDO
[b] ISRO
[c] CSIR
[d] ICAR
[e] NITI Aayog

Solution: [b]
www.anujjindal.in

Q8: With which organization has the Ministry of Social Justice and Empowerment joined
hands to introduce a transformative 70-hour interactive Employability Skills course for
Persons with Disabilities?
[a] Smile Foundation
[b] Tata Trusts
[c] Reliance Foundation
[d] Azim Premji Foundation
[e] Enable India

Solution: [e]
www.anujjindal.in

Q9: NITI Aayog in collaboration with the Ministry of Health & Family Welfare and which
State Government recently organized a National Workshop on "Strengthening Mental
Health Services and Care in India"?
[a] Uttar Pradesh
[b] Karnataka
[c] Maharashtra
[d] Rajasthan
[e] Gujarat

Solution: [b]
www.anujjindal.in

Q10: Which state has hosted 27th National Youth Festival under the theme “Viksit
Bharat@ 2047: युवा के लिए, युवा के द्वारा”?
[a] Maharashtra
[b] Gujarat
[c] Rajasthan
[d] Tamil Nadu
[e] Madhya Pradesh

Solution: [a]
www.anujjindal.in

Q11: With which company has India Post Payments Bank recently signed an MoU to
foster economic growth and bring financial inclusion in the lives of rural community
beneficiaries under the CSR program?
[a] Hindustan Zinc Limited
[b] Google India
[c] Coal India Limited
[d] Amazon India
[e] Microsoft India

Solution: [a]
www.anujjindal.in
PIB
24*7
12th – 13th January 2024
www.anujjindal.in

Q1: Which ministry/ies have developed Panchayat Mausam Sewa Portal to provide
weather forecasts in English, Hindi, and 12 regional languages to every panchayat head
and panchayat secretary to reach every village of the country?
[a] Ministry of Earth Sciences
[b] Ministry of Rural Development
[c] Ministry of Panchayati Raj
[d] A and B
[e] A and C

Solution: [e]
www.anujjindal.in

IMD launches various


initiatives
India Meteorological Department (IMD)
celebrated the 150th Year of its establishment
and service to the nation and launched
various initiatives.
www.anujjindal.in

Panchayat Mausam Sewa Portal

• Jointly developed by: India Meteorological Department,


Ministry of Earth Sciences, Ministry of Panchayati Raj and Green
Alert Mausam Sewa

• It will provide weather forecasts in English, Hindi, and 12


regional languages to every panchayat head and panchayat
secretary to reach every village in the country.

• It will enrich every farmer with weather alerts, warnings,


medium range weather forecast to help them in planning the
agricultural activities.
www.anujjindal.in

National Framework of Climate


Services
• Objective: To strengthen the production, availability, delivery,
and application of science-based climate monitoring and
prediction services.

• It will mitigate climate risks for key sectors such as Disaster risk
reduction, Agriculture and food security, Water resources, Public
health, and Energy.

• It is premised on the Global Framework for Climate Services


(launched by the World Meteorological Organization).
www.anujjindal.in

Weather Analysis and Forecast


Enabling System (WAFES)
• It is an indigenously developed in-house Web-GIS-based
integrated Decision Support System.
• It is inspired by Pancha Mahabhuta i.e. Water, Air, Fire, Earth
and Sky.
• It is a visualization platform to analyze meteorological
observations and prediction models, aiding decision-making for
severe weather phenomena and their socio-economic impact.
• It will provide real-time information for various sectors such as
Urban, Power, Hydrology, Health, Energy, Agriculture, Transport
and Tourism.
www.anujjindal.in

MAUSAM mobile app

• It is an integrated GIS-based interactive mobile app for all


weather-related services like current weather, weather forecast
every hour to 7 days
• It will provide forecasts on an hourly, 3-hourly, and 6-hourly
basis up to the next 10-days, covering crucial weather
parameters.
• Through it, users can view observations, forecasts, and warnings
for their location through a map or search function using place
names, pin codes, or coordinates.
• It supports 12 Indian languages for varied users in India.
www.anujjindal.in

Q2: The National Highways Authority of India (NHAI) has announced the ‘One Vehicle,
One FASTag’ Initiative aimed to discourage the use of single FASTag for multiple vehicles
or linking multiple FASTags to a particular vehicle. Under it, FASTags with incomplete KYC
will get deactivated/blacklisted by banks post ___________. Fill the gap
[a] 31st January 2024
[b] 1st March 2024
[c] 31st April 2024
[d] 15th August 2024
[e] 10th October 2024

Solution: [a]
www.anujjindal.in

One Vehicle One FASTag


Initiative
What: The National Highways Authority
of India (NHAI) has announced the 'One
Vehicle, One FASTag' Initiative.
Why: To enhance the efficiency of the
Electronic Toll Collection system and
provide seamless movement at the Toll
Plazas.
www.anujjindal.in

About One Vehicle One FASTag


Initiative
• Objective: To discourage use of single FASTag for multiple
vehicles or linking multiple FASTags to a particular vehicle.

• It encourages FASTag users to complete ‘Know Your Customer’


(KYC) process of their latest FASTag as per RBI guidelines.

• It will come into effect from 31st January 2024.


www.anujjindal.in

Q3: With which country has India’s Khanij Bidesh India Limited (KABIL) signed the first-
ever agreement for the lithium exploration and mining project?
[a] Australia
[b] Chile
[c] Bolivia
[d] Argentina
[e] Peru

Solution: [d]
www.anujjindal.in

India-Argentina Agreement
for Lithium Exploration &
Mining
What: India’s Khanij Bidesh India
Limited (KABIL) has signed an
agreement with Argentina’s state-owned
enterprise of Catamarca province
Why: To start the exploration and
development of Five Lithium Blocks in
Argentina’s Catamarca province.
www.anujjindal.in

About Agreement
• KABIL will start the exploration and development of 5 lithium
brine blocks covering an area of about 15,703 Hectare, located
in the Catamarca province of Argentina.

• It will also set up a branch office in Catamarca, Argentina.

• Project cost: Rs 200 crores.


www.anujjindal.in

Q4: What is the name of Startup India’s flagship Accelerator Program launched recently
for extensive handholding of startups at the scaleup stage?
[a] Startup Help
[b] Startup Shala
[c] Startup Support
[d] Startup Mela
[e] Startup Induction

Solution: [b]
www.anujjindal.in

Startup Shala
What: Startup Shala has been launched
recently on the sidelines of the Startup India
Innovation Week being held from 10th-18th
January 2024.
Why: To drive startups towards success,
emphasizing specific sectors with personalised
support
www.anujjindal.in

About Startup Shala


• It is Startup India’s flagship Accelerator Program launched for
extensive handholding of startups at the scaleup stage.

• It is a 3-month long accelerator program for early-stage startups


to provide them access to knowledge, network, funds, or
guidance required to scale up.

• It will provide the essential resources, mentorship, and


networking opportunities to accelerate your startup's journey to
the next level.

• Each of its program cohorts will focus on a particular sector, the


first one being the clean-tech sector.
www.anujjindal.in

Q5: The sixth edition of the Youth Co: Lab National Innovation Dialogue 2023-2024 has
been recently launched in India. Which of the following statement/s is/are correct about
Youth Co: Lab?
[a] Its idea was co-created in 2017 by UNDP and the Citi Foundation
[b] It was launched in India in 2019 jointly by Atal Innovation Mission, NITI Aayog and
UNDP India
[c] It is a multi-dimensional and multi-level response to tackle challenges faced by young
people and enable them to find solutions to global challenges tackling SDGs
[d] A and B
[e] A, B and C

Solution: [e]
www.anujjindal.in

6th Youth Co: Lab


What: The sixth edition of the Youth
Co:Lab National Innovation Dialogue
2023-2024 has been launched in India
Why: To Support 50 Youth-led Social
Start-ups in Accelerating their
Businesses
www.anujjindal.in

About Youth Co:Lab


• Objective: To establish a common agenda for Asia-Pacific
countries to invest in and empower youth to accelerate the
implementation of the SDGs.

• Launch Year: 2019.


▪ It was co-created by UNDP and the Citi Foundation.

• It seeks to build responses to tackle challenges faced by young


people by bringing them to forefront to find solutions to global
challenges in tackling SDGs.

• It was launched in collaboration with Atal Innovation Mission,


NITI Aayog.
www.anujjindal.in

Themes of 6th Youth Co:Lab


• Interested startups can submit their application under the
themes namely:

▪ Agriculture Fintech and Insurtech for small and marginal


farmers

▪ Innovative solutions for Indigenous groups, tribal


communities

▪ Assistive technology and solutions


www.anujjindal.in

Q6: Which Ministry has introduced the ANUBHAV Awards Scheme to recognize the
contribution made by Retired Officials to nation-building while working in Government
and to document the administrative history of India by written narratives?
[a] Ministry of Education
[b] Ministry of Defence
[c] Ministry of Home Affairs
[d] Ministry of Personnel, Public Grievances & Pensions
[e] Ministry of Social Justice and Empowerment

Solution: [d]
www.anujjindal.in

ANUBHAV Awards Scheme


2024
Ministry of Personnel, Public Grievances
& Pensions has launched the ANUBHAV
Awards Scheme.
www.anujjindal.in

About ANUBHAV Awards Scheme


• Objectives:
▪ To recognize the contribution made by Retired Officials to
nation-building while working in Government
▪ To document the administrative history of India through
written narratives.
• To participate in it, retiring Central Government
employees/pensioners are required to submit their Anubhav
write-ups, 8 months before retirement and up to 1 year after
their retirement.
• Thereafter, the write-ups are published after assessment by
concerned Ministries/Departments.
• The published write-ups will be shortlisted for Anubhav Awards
and Jury Certificates.
www.anujjindal.in

About ANUBHAV Portal


• It was launched by the Ministry of Personnel, Public Grievances
& Pensions in 2015.

• It is for sharing experiences of retiring/retired Central


Government employees while working with the Government.
www.anujjindal.in
www.anujjindal.in

Q7: Ayushman Bharat Pradhan Mantri - Jan Arogya Yojana (AB PM-JAY) has recently
crossed the milestone of issuing 30 crore Ayushman cards. Which State tops the list of
states with the highest number of Ayushman Cards created?
[a] Uttar Pradesh
[b] Maharashtra
[c] Bihar
[d] Madhya Pradesh
[e] Rajasthan

Solution: [a]
www.anujjindal.in

Q8: India’s First National Highway Steel Slag Road section has been inaugurated recently
on National Highway (NH) – 66. NH–66 connects
[a] Nagpur-Bhopal
[b] Mumbai-Goa
[c] Surat-Jaipur
[d] Chandigarh-Delhi
[e] Patna-Kolkata

Solution: [b]
www.anujjindal.in

Q9: NITI Aayog in collaboration with which organization has launched Compendium of
Inspiring Stories on Millets Mainstreaming in India, Asian and African countries?
[a] UNEP
[b] UNDP
[c] UNWFP
[d] UNESCAP
[e] UNMEGP

Solution: [c]
www.anujjindal.in

Q10: How much amount has been released as the first installment in Pradhan Mantri
Janjati Adivasi Nyaya Maha Abhiyan (PM JANMAN) to PVTG beneficiaries across 19 States
and UT, for the construction of 1 lakh pucca houses under Pradhan Mantri Awas Yojana -
Gramin?
[a] Rs. 330 crores
[b] Rs. 450 crores
[c] Rs. 540 crores
[d] Rs. 660 crores
[e] Rs. 780 crores

Solution: [c]
www.anujjindal.in
PIB
24*7
17th – 18th January 2024
www.anujjindal.in

Q1: Which Ministry has recently launched the MPLADS e-SAKSHI Mobile Application to
track the fund flow under the Member of Parliament Local Area Development Scheme-
2023?
[a] Ministry of Parliamentary Affairs
[b] Ministry of Housing and Urban Affairs
[c] Ministry of Statistics & Programme Implementation
[d] Ministry of Home Affairs
[e] Ministry of Finance

Solution: [c]
www.anujjindal.in

MPLADS e-SAKSHI Mobile


Application
What: The Ministry of Statistics & Programme
Implementation has launched the MPLADS e-
SAKSHI Mobile Application.
Why: For revising fund flow procedure under
the Members of Parliament Local Area
Development Scheme (MPLAD) Scheme
www.anujjindal.in

About e-SAKSHI Mobile App


• Objective: To enable the MPs to recommend works of
developmental nature for the creation of durable community
assets based on locally felt needs.

• It will offer convenience and accessibility to MPs to propose,


track, and oversee the projects at their fingertips.

• It will enable MPs to engage with and manage development


projects in their constituencies.
www.anujjindal.in

About Members of Parliament


Local Area Development Scheme
• Objective: To enable MPs to recommend works of a
developmental nature with emphasis on the creation of durable
community assets in their Constituencies.

• Launch Year: 1993

• Implementing Ministry: Ministry of Statistics & Programme


Implementation

• Funding: It is a Central Sector Scheme (fully funded by the


Central Government)
www.anujjindal.in

About Members of Parliament


Local Area Development Scheme
• Under it
▪ MPs have the choice to suggest District Collector for works
to the tune of Rs.5 Crores per annum to be taken up in their
constituency.
▪ Rajya Sabha MPs can recommend works in one or more
districts in the State from where they are elected.
▪ Nominated Members of Lok Sabha and Rajya Sabha may
select any one or more Districts from any one State for
implementation of their choice of work.

• Annual fund entitlement per MP per constituency: Rs.5 crore.


• Minimum amount per project: Rs. 1 Lakh.
www.anujjindal.in

Q2: What is the central theme of the 9th edition of the India International Science
Festival (IISF) 2023 being held in Faridabad, Hyderabad?
[a] Addressing Global Challenges through Science and Technology
[b] Science and Technology for 4th Industrial Revolution
[c] India’s Innovation for World
[d] Celebrating Creativity in Science, Technology, and Innovation for Prosperous India
[e] Science and Technology Public Outreach in Amrit Kaal

Solution: [e]
www.anujjindal.in

India International Science


Festival 2023
What: The 9th edition of the India
International Science Festival (IISF) 2023
is being held at Faridabad, Haryana at the
Campus of Translational Health Science
and Technology Institute and Regional
Centre for Biotechnology.
www.anujjindal.in

About IISF
• Objective: To commemorate the world of science, making it
accessible to all.

• It is the largest science festival in India organized since 2015.

• It is a collaborative endeavor between the Ministry of Science


and Technology, the Ministry of Earth Sciences, the Department
of Space, and the Department of Atomic Energy in partnership
with Vijnana Bharati.
www.anujjindal.in

About 9th IISF 2023


• Objectives:
▪ To celebrate the achievements of India in the frontier areas
of science and technology.
▪ To acknowledge the achievements of science enthusiasts
and inculcate scientific temper among young students and
Indian citizens.
• Central theme: “Science and Technology Public Outreach in
Amrit Kaal”.
• Organiser: Department of Science and Technology
▪ Coordinating and Implementation body: National Innovation
Foundation-India
▪ Host Institute: Department of Biotechnology institutes -
Regional Centre for Biotechnology – Translational Health
Science and Technology Institute located in Faridabad.
www.anujjindal.in

Q3: Which PSU became the first in India to issue Japanese-denominated Yen Green
Bonds to finance eligible Green Projects?
[a] REC Limited
[b] BHPCL
[c] ONGC Limited
[d] PFC
[e] NHPC

Solution: [a]
www.anujjindal.in

First Yen Green Bonds


What: REC Limited became the first PSU to
successfully issue Japanese-denominated Yen
Green Bonds under its US$ 10 billion Global
Medium Term Notes Program

Why: To finance the Eligible Green Projects


under the Company’s Green Finance
Framework, RBI’s External Commercial
Borrowings Guidelines etc.
www.anujjindal.in

About Yen Green Bonds

• Proceeds from these bonds will be used to finance the Eligible


Green Projects

• Tenor/Interest: 5-year, 5.25-year and 10-year bonds issued at


yield of 1.76%, 1.79% and 2.20% respectively.
www.anujjindal.in

About REC Limited


• It is a 'Maharatna' company under the Ministry of Power.
• It is registered with RBI as Non-Banking Finance Company, Public
Financial Institution, and Infrastructure Financing Company.
• It finances the entire Power-Infrastructure sector comprising
Generation, Transmission, Distribution, Renewable Energy and
new technologies
• More recently, it has also diversified into the non-power
Infrastructure sector comprising roads/expressways, metro rail,
airports, Social and Commercial Infrastructure, etc.
• It provides loans of various maturities to State, Central, and
Private Companies for the creation of infrastructure assets in the
country.
www.anujjindal.in

Q4: Which of the following statements are correct about the Group
Accident Insurance Scheme for fisherman?
(1) It was launched in 2020-21 as a sub-component of Pradhan Mantri
[a] 3, 4 and 5 Only
Matsya Sampada Yojana
[b] 1, 2 and 3 Only
(2) It offers insurance coverage of Rs. 10 lakhs against accidental death or
Permanent Total Disability [c] 2 and 3 Only

(3) It provides insurance coverage to men/women fishers in the age group [d] 1 and 4 Only
from 18 to 55 years [e] 2, 3 and 5 Only
(4) It provides insurance coverage only for a period of 12 months on an
annual premium payment
(5) To avail of benefits beneficiary men/women fishers must pay 10% of
premium annually

Solution: [d]
www.anujjindal.in

Group Accident Insurance


Scheme
What: The Ministry of Fisheries, Animal
Husbandry & Dairying has organized the
National Conference on Fisheries and
Aquaculture Insurance in hybrid mode at
PUSA, New Delhi.

Why: To offer a platform to all stakeholders


from the insurance sector to deliberate and
discuss ways to enhance access to insurance
schemes including Group Accident Insurance
Scheme (GAIS)
www.anujjindal.in

About Group Accident Insurance


Scheme
• Objective: To offer insurance to fishers including fishermen,
fisherwomen, fish workers, fish farmers, and others directly
involved in fishing and fisheries-related allied activities.
• Launch Year: 2020-21
• Parent Scheme: It is a sub-component of under Pradhan Mantri
Matsya Sampada Yojana (PMMSY).
• Type: Centrally sponsored scheme
• Nodal implementing agency: National Fisheries Development
Board (NFDB)
▪ Insurance claims are managed by Providence India Insurance
Broking Pvt ltd as an intermediary through Oriental
Insurance Company Limited (OICL).
www.anujjindal.in

About Group Accident Insurance


Scheme
• Beneficiary: It insures men/women fishers in the age group from
18 to 70 years as approved by the State/UT fisheries
department.

• Benefits: It offers insurance coverage of


▪ Rs. 5.00 lakh against accidental death or Permanent Total
Disability
▪ Rs. 2.50 lakh against Permanent Partial Disability
▪ Rs. 25,000 against accidental hospitalization
www.anujjindal.in

About Group Accident Insurance


Scheme
• Duration of Insurance cover and Premium: It is for a period of 12
months and premium shall be paid annually.
▪ No beneficiary contribution is required as the premium is
fully funded by the Central and respective State/UT
Governments as per the funding pattern of the PMMSY.
✓ 60:40 between the Central and General State
Government
✓ 90:10 between the Central and North Eastern and
Himalayan states
✓ 100% Central for UTs
www.anujjindal.in

Q5: Which of the following states are the best performers in


Category A (States with a population greater than 1 crore) of
the 4th States Startup Ranking 2022 released recently by
DPIIT?
(1) Gujarat [a] 2, 3, 4 and 5

(2) Maharashtra [b] 1, 2, 3 and 4


[c] 2, 3 and 5
(3) Tamil Nadu
[d] 1, 3, 4 and 5
(4) Karnataka
[e] All of the above
(5) Kerala

Solution: [d]
www.anujjindal.in

States Startup Ranking


Framework 2022 Results
What: On the occasion of 8th National Startup
Day 2024 (on January 16, 2024), the DPIIT
under Startup India has announced the results
of

• 4th edition of ‘States’ Startup Ranking 2022-


On Support to Startup Ecosystems of
States and Union Territories

• 4th ‘National Startup Awards 2023-


Celebrating Achievers Ahead of the Curve’
www.anujjindal.in

About 4th States Startup Ranking


2022
• 33 States/UTs were evaluated across five performance
categories viz. best performers, top performers, leaders, aspiring
leaders, and emerging startup ecosystems.

• Its evaluation is focused on 7 broad Reform Areas with 25 Action


Points, contributing to a maximum score of 100 marks, including
feedback.
▪ The reform areas are Institutional Support, Fostering
Innovation and entrepreneurship, Access to Market,
Incubation, Mentorship Support, Funding Support, Capacity
Building of Enablers, Roadmap to a Sustainable Future.
www.anujjindal.in

About 4th States Startup Ranking


2022

• States were ranked based on initiatives for developing startup


ecosystems, categorized by population size i.e. Category A & B.

▪ Category A: It included States and UTs with a population of


more than 1 crore.

▪ Category B: It included States and UTs with a population of


less than 1 crore.
www.anujjindal.in

Category A States (Population more than 1 crore)


Best performer • Gujarat
• Karnataka
• Kerala
• Tamil Nadu
Top Performer • Maharashtra
• Odisha
• Punjab
• Rajasthan
• Telangana
Leader • Andhra Pradesh
• Assam
• Madhya Pradesh
• Uttar Pradesh
• Uttarakhand
Aspiring Leader • Bihar
• Haryana
Emerging Ecosystems • Chhattisgarh
• Delhi
• Jammu and Kashmir
www.anujjindal.in

Category B States (Population less than 1 crore)

Performance States/UTs
Best performer • Himachal Pradesh
Top Performer • Arunachal Pradesh
• Meghalaya
Leader • Goa
• Manipur
• Tripura
Aspiring Leader • Andaman and Nicobar Islands
• Nagaland
Emerging Ecosystems • Chandigarh
• Dadra and Nagar Haveli and Daman and Diu
• Ladakh
• Mizoram
• Puducherry
• Sikkim
www.anujjindal.in
www.anujjindal.in

Q6: With which organization has the Department of Empowerment of Persons with
Disabilities signed an MoU to aggregate employment opportunities for Divyangjans
through the innovative PM-DAKSH-DEPWD digital portal?
[a] IRDA
[b] CSIR
[c] ESIC
[d] NHRDN
[e] REC

Solution: [d]
www.anujjindal.in

Q7: Government has set a target to reduce Accident Deaths by __________ by 2030. Fill
the gap
[a] 70%
[b] 50%
[c] 60%
[d] 40%
[e] 30%

Solution: [b]
www.anujjindal.in

Q8: Keeping in view India’s vision of becoming ‘Atmanirbhar’, Production Linked


Incentive (PLI) Schemes for ________ key sectors with an incentive outlay of
___________ are under implementation to enhance India’s Manufacturing capabilities
and Exports. Fill the gaps
[a] 16, US $30 billion
[b] 14, US $26 billion
[c] 18, US $16 billion
[d] 20, US $23 billion
[e] 22, US $30 billion

Solution: [b]
www.anujjindal.in

Q9: Which Ministry has developed India Digital Ecosystem Architecture (InDEA) 2.0 to
create a unified digital ecosystem for various government services and initiatives?
[a] Ministry of Science and Technology
[b] Ministry of Education
[c] Ministry of Commerce and Industry
[d] Ministry of Health and Family Welfare
[e] Ministry of Electronics and Information Technology

Solution: [e]
www.anujjindal.in

Q10: Which Ministry and Kerala Government along with Tata Steel Limited have
established India’s first Graphene Centre at Makers Village Kochi?
[a] Ministry of Science and Technology
[b] Ministry of Mines
[c] Ministry of Electronics and Information Technology
[d] Ministry of Defence
[e] Ministry of Commerce and Industry

Solution: [c]
PIB
24*7
19th – 20th January 2024
www.anujjindal.in

Q1: Identify incorrect statement about the 3rd Advance Estimates of Area and
Production of Various Horticultural Crops for the year 2022-23, recently released by the
Ministry of Agriculture and Farmers’ Welfare.
[a] Total horticulture production for the year 2022-23 is 355.25 million tonnes, a decrease
of 2.32% compared to the year 2021-22
[b] Fruit production is estimated to increase to 109.53 million tonnes in the year 2022-23
from 107.51 million tonnes in the year 2021-22
[c] Production of vegetables is estimated to be 213.88 million tonnes in the year 2022-23
[d] Production of plantation crops is estimated to increase to 16.84 million tonnes in the
year 2022-23
[e] Potato production is expected to be 60.22 million tonnes in the year 2022-23

Solution: [a]
www.anujjindal.in

3rd Advance Estimate -


Horticultural Crops
The Ministry of Agriculture and Farmers’
Welfare has released the Third advance
estimate of area and production of
various horticultural crops for the year
2022-23.
www.anujjindal.in

Highlights of the Estimate

• Total horticulture production: 355.25 million tonnes


▪ It is about 8.07 million tonnes more (an increase of 2.32%) than
the year 2021-22

• Total Area under production: 28.34 million hectares

• Fruit production: 109.53 million tonnes


▪ It was 107.51 million tonnes in 2021-22.

• Production of vegetables: 213.88 million tonnes


▪ It was 209.14 million tonnes.
www.anujjindal.in

Highlights of the Estimate

• Production of plantation crops: 16.84 million tonnes.


▪ It was 15.76 million tonnes in 2021-22 i.e. an increase of about
6.80%.

• Potato production: 60.22 million tonnes


▪ It was 56.18 million tonnes in 2021-22.

• Tomato production: 20.37 million tonnes


▪ It was 20.69 million tonnes in 2021-22.
www.anujjindal.in

Q2: Which of the following statements are incorrect


about the recently launched Alliance for Global Good-
Gender Equity and Equality? [a] 2 and 3 Only
(1) It was launched by France at the 2024 Gender Equality [b] 1 and 2 Only
Summit
[c] 1, 2 and 3 Only
(2) India’s NITI Aayog is the institutional partner of it
[d] 2, 3 and 4 Only
(3) World Economic Forum is its network partner
[e] All of the above
(4) It is supported by the Bill and Melinda Gates
Foundation

Solution: [b]
www.anujjindal.in

Alliance for Global Good-


Gender Equity and Equality
What: India has launched ‘Alliance for
Global Good- Gender Equity and
Equality’ at the World Economic Forum,
Annual Meeting held in Davos,
Switzerland.
www.anujjindal.in

About Alliance for Global Good-


Gender Equity and Equality
• Objective: To bring together global best practices, knowledge
sharing, and investments in the identified areas of women’s
health, education, and enterprise.

• Supported by: Bill and Melinda Gates Foundation

• Housed and anchored by: CII Centre for Women Leadership

• Network Partner: World Economic Forum

• Institutional Partner: Invest India


www.anujjindal.in

Q3: President of India Droupadi Murmu will be conferring the Pradhan Mantri Rashtriya
Bal Puraskar 2024 to 19 exceptional children. Which of the following statement is
incorrect about the award?
[a] It is awarded to Indian citizen children for their exceptional achievement
[b] It is conferred upon the children in the age group 10 – 18 years
[c] It is awarded in the seven categories
[d] Ministry of Women and Child Development is the nodal ministry for this award
[e] Each awardee is given a medal and a Certificate

Solution: [b]
www.anujjindal.in

Pradhan Mantri Rashtriya


Bal Puraskar 2024
What: President of India Droupadi
Murmu will be conferring the Pradhan
Mantri Rashtriya Bal Puraskar, 2024 to
19 exceptional children.
www.anujjindal.in

About Pradhan Mantri Rashtriya


Bal Puraskar 2024
• These 19 children have been selected for their exceptional
achievements in the fields of Art and Culture (7), Bravery (1),
Innovation (1), Science and Technology (1), Social Service (4),
and Sports (5).

• It comprises 9 boys and 10 girls among the awardees belonging


to 18 States and UTs including 2 Aspirational Districts.
www.anujjindal.in

About Pradhan Mantri Rashtriya


Bal Puraskar
• It is awarded to Indian citizen children for their exceptional
achievement.

• It is conferred upon the children in the age group 5 – 18 years,


for their excellence in seven categories.

• Seven Categories: Art and culture, Bravery, Environment,


Innovation, Science and Technology, Social Service, and Sports

• Nodal Ministry: Ministry of Women and Child Development

• Reward: Each awardee is given a medal and a Certificate.


www.anujjindal.in

Q4: The National Quantum Mission (NQM) was launched in 2023 with a total outlay of
__________ for a period of _________. Fill the gaps
[a] Rs. 10450.33 Crore, 10 years
[b] Rs. 5504.54 Crore, 5 years
[c] Rs. 6003.65 Crore, 8 years
[d] Rs. 8304.23 Crore, 15 years
[e] Rs. 12342.78 Crore, 12 years

Solution: [c]
www.anujjindal.in

National Quantum Mission


What: The 1st meeting of the Mission
Governing Board of the National
Quantum Mission was held recently in
New Delhi.
Why: To discuss the implementation
strategy and timelines of NQM as well as
the formation of the Mission
Coordination Cell.
www.anujjindal.in

About National Quantum Mission


• Objective: To seed, nurture and scale up scientific and industrial
R&D and create an innovative ecosystem in Quantum
Technology.

• Launch Year: 2023

• Total outlay: Rs.6003.65 Crore

• Duration: 8 years (2023-24 to 2030-31)

• Implementing Agency/Department: Department of Science and


Technology
www.anujjindal.in

Q5: Where is Asia’s largest event in the Civil Aviation sector - “Wings India 24” being
organized by the Ministry of Civil Aviation, the Airports Authority of India, and FICCI?
[a] Bengaluru
[b] Mumbai
[c] Hyderabad
[d] Chennai
[e] Lucknow

Solution: [c]
www.anujjindal.in

Aviation Expo - Wings India


2024
What: The largest event in the Civil
Aviation sector in Asia - “Wings India
24” is being organized in Begumpet
Airport in Hyderabad, Telangana.
www.anujjindal.in

About Wings India 2024


• Organisers: Ministry of Civil Aviation, Airports Authority of India
and FICCI

• Theme: “Connecting India to the World in Amrit Kaal: Setting


the Stage for India Civil Aviation @2047”

• Host State: Telangana


www.anujjindal.in

Major announcements made at


Wings India 2024
• Launch of UDAN 5.3: It includes routes that were discontinued
before completion of tenure/cancelled/terminated from past
UDAN rounds.
▪ Only Individual Route Proposals are being allowed for the
bidding process.

• Launch of the Airbus-Air India Training Centre in Gurugram

• Airbus manufacturing contracts with TATA ASL and Mahindra


Aerospace Structures Pvt. Ltd. to foster more pilots.
www.anujjindal.in
www.anujjindal.in

Q6: The 8th Meeting of the Mission Steering Group of National Technical Textiles
Mission (NTTM) recently approved 11 R&D projects valuing Rs. 103 Crores across
different areas of Technical Textiles. When was NTTM launched?
[a] 2018
[b] 2019
[c] 2020
[d] 2021
[e] 2022

Solution: [c]
www.anujjindal.in

Q7: The 2nd meeting of the Indo-French Joint Committee of Science and Technology
recently took place in New Delhi to discuss the ways of catalyzing a stronger and
rejuvenated Indo-French scientific partnership. When was 1st JCST held?
[a] 2015
[b] 2018
[c] 2019
[d] 2021
[e] 2023

Solution: [b]
www.anujjindal.in

Q8: What is the name of the Science Literature Festival being organized as a part of the
India International Science Festival 2023?
[a] Vigyanika
[b] Takaneekee
[c] Gyanika
[d] Yuva Vigyan
[e] Bhramand

Solution: [a]
www.anujjindal.in

Q9: Knowledge and Awareness Mapping Platform (KAMP) is an initiative of which


organization to develop creativity, meaningful learning, and critical thinking skills in
students?
[a] ISRO
[b] DRDO
[c] CSIR
[d] ICAR
[e] NCERT

Solution: [c]
www.anujjindal.in

Q10: Which PSU has been awarded under the ‘Financial Services Sector (Other than
Banking and Insurance)’ category at the ICAI Awards for Excellence in Financial Reporting
for FY 2022-23?
[a] ONGC
[b] PFC
[c] NHPC
[d] SECI
[e] REC

Solution: [e]
PIB
24*7
21st – 22nd January 2024
www.anujjindal.in

Q1: Prime Minister Narendra Modi recently announced the launch of Pradhan Mantri
Suryodaya Yojana (PSY) with target to install rooftop solar power systems at
__________ households across the country. Fill the gap
[a] 10 million
[b] 15 million
[c] 20 million
[d] 25 million
[e] 30 million

Solution: [a]
www.anujjindal.in

Pradhan Mantri Suryodaya


Yojana
What: Prime Minister Narendra Modi has
announced Pradhan Mantri Suryodaya
Yojana
www.anujjindal.in

About Pradhanmantri Suryodaya


Yojana
• Objective: To provide electricity to low and middle-income
individuals through solar rooftop installations, along with
offering additional income for surplus electricity generation.

• Target: Installing rooftop solar on 1 crore (10 million) houses.

• It will help to reduce the electricity bill of the poor and


middle class and also make India self-reliant in the energy
sector.
www.anujjindal.in

Background

• Rooftop Solar Programme: It was launched in 2014 with an


aim to achieve a cumulative installed capacity of 40,000
megawatts (MW) or 40 gigawatts (GW) by 2022. However,
this target couldn’t be achieved.
▪ Phase 2: As a result, the government had extended the
deadline from 2022 to 2026

• Pradhan Mantri Suryodaya Yojana seems to be a new attempt


to help reach the target of 40 GW rooftop solar capacity.
www.anujjindal.in

Q2: Which of the following statements are incorrect about recently approved
New Solar Power Scheme for Particularly Vulnerable Tribal Groups (PVTG)
Habitation/Villages under Pradhan Mantri Janjati Adivasi Nyaya Abhiyan (PM-
JANMAN)?
(1) It aims to electrify 2 lakh un-electrified households in PVTG areas in 18 [a] 1, 2 and 3 Only
States/UTs identified by the Ministry of Tribal Affairs [b] 2, 3 and 4 Only
(2) It will be implemented for period of 5 years with an outlay of Rs. 515 crores [c] 3, 4 and 5 Only
(3) It has two components viz. Electrification of PVTG households through Off- [d] 1, 2, 4 and 5 Only
grid solar power and Solarization of multi-purpose centers in PVTG areas
[e] 2, 3, 4 and 5 Only
(4) Solarization of multi-purpose centers in PVTG areas component has financial
outlay of Rs. 500 crores
(5) Rs. 1,00,000 per PVTG households will be provided under the electrification
of PVTG households through Off-grid solar power component

Solution: [d]
www.anujjindal.in

New Solar Power Scheme


for PVTGs
What: President has accorded approval
for the implementation of a New Solar
Power Scheme for Particularly
Vulnerable Tribal Groups (PVTG)
Habitation/Villages under Pradhan
Mantri Janjati Adivasi Nyaya Abhiyan
(PM-JANMAN).
www.anujjindal.in

About New Solar Power Scheme


• Objective: To cover the electrification of 1 lakh un-electrified
households in PVTG areas identified by the Ministry of Tribal
Affairs
• Coverage: 18 States/UTs
• Timeline: 3 years (2023-24 to 2025-26)
• Funding Outlay: Rs 515 crore.
▪ The funds will be sourced from Ministry of New and
Renewable Energy's Development Action Plan for Scheduled
Tribes
▪ It will be disbursed over 3 years
Year 2023-24 2024-25 2025-26 Total
Fund Rs. 20 Rs. 255 Rs. 240 Rs. 515
Allocated crores crores crores crores
www.anujjindal.in

About New Solar Power Scheme


• Two components:
▪ Electrification of 1 Lakh PVTG households through Off-grid
solar power: It will comprise Solar Home Lighting System for
scattered unelectrified HHs in PVTG areas and Solar Mini-
grids for cluster of HHs in a PVTG habitation/ hamlet.
✓ Financial Outlay: Rs. 500 Crores (100% Central Share)
✓ Rs. 50,000 per HHs will be provided

▪ Solarization of multi-purpose centres (MPCs) in PVTG areas


where grid-electricity is not available: It will be achieved by
installation of Off-grid Solar power pack with battery bank.
✓ Financial Outlay: Rs. 15 Crores (100% Central Share)
✓ Rs. 1 lakh per MPC will be provided
www.anujjindal.in

About New Solar Power Scheme

• Implementing agency: Respective DISCOMs in PVTG area.

• Monitoring: Ministry of New and Renewable Energy and


Ministry of Tribal Affairs
www.anujjindal.in

Q3: Identify incorrect statement with regard to findings of NITI Aayog’s Discussion Paper
‘Multidimensional Poverty in India since 2005-06’
[a] Poverty Headcount Ratio has declined from 29.17% in 2013-14 to 11.28% in 2022-23 i.e. a
reduction of 17.89 percentage points
[b] Pace of decline in poverty headcount ratio using exponential method was much faster
between 2015-16 to 2019-21 compared to 2005-06 to 2015-16
[c] Largest decline in poverty was seen in Bihar, where 5.94 crore people have escaped
multidimensional poverty during the last 9 years
[d] In terms of indicator wise deprivation, lowest levels were seen Child and adolescent
mortality (2.06%), electricity (3.27%) and bank account (3.69%)
[e] None of the above

Solution: [c]
www.anujjindal.in

NITI Aayog Discussion


Paper on Multidimensional
Poverty in India
What: According to NITI Aayog’s
Discussion Paper ‘Multidimensional
Poverty in India since 2005-06’, 24.82
crore people in India have escaped
multidimensional poverty in last nine
years (2013-14 to 2022-23).
www.anujjindal.in

About National Multidimension


Poverty Index
• Developed by: NITI Aayog with help of UNDP and Oxford Policy
and Human Development Initiative.

▪ 12 indicators: Nutrition, child and adolescent mortality,


maternal health*, years of schooling, school attendance,
cooking fuel, sanitation, drinking water, electricity, housing,
assets, and bank accounts*.

• It uses the Alkire Foster methodology to assess the decline in


poverty rates.
www.anujjindal.in

Highlights of Discussion Paper


• Poverty Headcount Ratio (HCR): It has declined from 29.17% in
2013-14 to 11.28% in 2022-23 i.e. a reduction of 17.89
percentage points.
• Pace of decline in poverty headcount ratio: It was much faster
between 2015-16 to 2019-21 (10.66% annual rate of decline)
compared to period 2005-06 to 2015-16 (7.69% annual rate of
decline).
www.anujjindal.in

Highlights of Discussion Paper


• Largest decline registered in
▪ Uttar Pradesh (5.94 crore people escaping multidimensional
poverty during the last 9 years).
▪ Bihar (3.77 crore)
▪ Madhya Pradesh (2.30 crore)
▪ Rajasthan (1.87 crore)
• Indicator wise Performance: All 12 indicators of National MPI
have recorded significant improvement during the entire study
period.
▪ Highest Deprivation levels: Cooking fuel (43.90%) and
housing (41.37%)
▪ Lowest deprivation levels: Child and adolescent mortality
(2.06%), electricity (3.27%) and bank account (3.69%).
www.anujjindal.in

Highlights of Discussion Paper


• Significance: India is likely to achieve Sustainable Development
Goal (SDG) Target 1.2 (reducing multidimensional poverty by at
least half by 2023) well before target year of 2030.
www.anujjindal.in
www.anujjindal.in

Q4: The average daily availability of power in rural areas has gone up from 12.5
hours in 2015-16 to about ______________ now, and in urban areas from around 20-
21 hours to ________. Energy shortage has come down from 4.5% in 2014 to less
than _________. Fill the gaps
[a] 22 hours, 22.2 hours, 3%
[b] 21 hours, 23.8 hours, 1%
[c] 23 hours, 22.9 hours, 2%
[d] 20 hours, 23.2 hours, 3%
[e] 19 hours, 23.7 hours, 2.3%

Solution: [b]
www.anujjindal.in

Q5: Government of India has set ambitious goals of installing 500 GW non-fossil
electricity capacity by 2030. As of December 31, 2023, the installed capacity from
non-fossil fuel sources was ________________. This accounted for a significant
_________ of total installed generation capacity. Fill the gaps
[a] 230.5 GW, 47%
[b] 190.3 GW, 42%
[c] 179.6 GW, 39%
[d] 210.2 GW, 55%
[e] 187.5 GW, 44%

Solution: [e]
www.anujjindal.in

Q6: With which bank has Indian Renewable Energy Development Agency Ltd.
(IREDA) recently signed MoU for co-lending in Renewable Energy Projects?
[a] ICICI Bank
[b] Indian Overseas Bank
[c] State Bank of India
[d] HDFC Bank
[e] Canara Bank

Solution: [b]
www.anujjindal.in

Q7: Which Central Armed Police Force became the first to establish Cyber Security
Operations Centre at its headquarters?
[a] Border Security Force
[b] Central Reserve Police Force
[c] Central Industrial Security Force
[d] Sashastra Seema Bal
[e] Assam Rifles

Solution: [e]
www.anujjindal.in

Q8: With which organisation has the Union Cabinet recently approved MoU to
strengthen cooperation in semiconductor for the advancement of Industries and
digital technologies?
[a] ASEAN
[b] EU
[c] NAFTA
[d] GCC
[e] RCEP

Solution: [b]
www.anujjindal.in

Q9: The Union Cabinet has recently approved MoU between India and which
country on cooperation in the field of sharing successful Digital Solutions
implemented at Population Scale for Digital Transformation?
[a] South Africa
[b] Vietnam
[c] Thailand
[d] Chile
[e] Kenya

Solution: [e]
www.anujjindal.in

Q10: With which countries has Union Cabinet recently


approved signing MoUs in the field of cooperation in the
field of Medical Products Regulation?
(1) Netherlands [a] 3, 4 and 5 Only
(2) Chile [b] 2 and 5 Only
(3) Ecuador [c] 3 and 4 Only
(4) Peru [d] 4 and 5 Only
(5) Dominican Republic [e] 3 and 5 Only

Solution: [e]
PIB
24*7
23rd – 24th January 2024
www.anujjindal.in

Annual Status of Education


Report (ASER) 2023 ‘Beyond
Basics’
What: The Annual Status of Education
Report (ASER) 2023 titled ‘Beyond
Basics’ has been released by the
Pratham Foundation.
www.anujjindal.in

Introduction
• Annual Status of Education Report (ASER) is a nationwide
citizen-led household survey that provides a snapshot of the
status of children’s schooling and learning in rural India

• Launch Year: The first ASER was conducted in 2005 and since
then conducted annually.
• Conducted by: Pratham NGO.

• Methodology: It is a household-based rather than a school-


based survey.

▪ Household-based survey: It allows it to include out-of-school


children and children attending different types of schools.
www.anujjindal.in

Introduction
• Coverage: It was a nationwide citizen-led rural household
survey.
▪ Urban areas are not covered in it.

• Types of ASER Survey


▪ Basic ASER Survey: It tracks enrolment for children aged 3-16
and assesses basic reading and arithmetic of children aged 5-
16.
▪ ASER Beyond Basic Survey: It focuses on the activities,
awareness, abilities, and aspirations of youth aged 14 to 18
www.anujjindal.in

About ASER 2023 ‘Beyond Basics’ survey


• Coverage: It was conducted in 28 districts across 26 states,
reaching a total of 34,745 youth in the age group 14-18 years.
▪ One rural district was surveyed in each major state, except
Uttar Pradesh and Madhya Pradesh, where two rural
districts were surveyed.

• Focused Age Group: 14-18

• It looks ’beyond basics’ to explore a wider set of domains


beyond foundational skills
www.anujjindal.in

About ASER 2023 ‘Beyond Basics’ survey


• Methodology: It explored the following domains:

▪ Activity: What activities are India’s youth currently engaged


in?

▪ Ability: Do they have basic and applied reading and math


abilities?

▪ Awareness and Digital aptitude: Do they have access to


smartphones? What do they use smartphones for, and can
they do simple tasks on their smartphones?
www.anujjindal.in

Key Findings of ASER 2023 ‘Beyond Basics’


ACTIVITY

Enrolment Rates
• Overall, 86.8% of 14-18-year-olds are enrolled in an educational
institution.
▪ 53% are still in school (Std X or below)
▪ 28% are in either Std XI or XII.
▪ 7% are in college.
▪ 13% are currently not enrolled anywhere.

• Age 14: 72% of students are enrolled in govt institutions.

• Age 18: 44% of students are enrolled in govt institutions.


www.anujjindal.in

Key Findings of ASER 2023 ‘Beyond Basics’

• Most of the young people in the age group of 14-18 years were
enrolled in the Arts/Humanities stream.

▪ In Std XI or higher, more than half are enrolled in the


Arts/Humanities stream (55.7%).

▪ Females are less likely to be enrolled in the STEM stream


(28.1%) than males (36.3%).
www.anujjindal.in

Key Findings of ASER 2023 ‘Beyond Basics’

Currently Not Enrolled


• Age 14: % Youth not enrolled is low at 3.9%. There is hardly any
difference between boys and girls
• Age 18: % Youth not enrolled is much higher at 32.6%
▪ 33.4% of females were not enrolled.
▪ 31.6% of males were not enrolled

Vocational Training
• Only about 5.6% of all youth in the age group 14-18 are doing
vocational courses.
• Most of those who do such courses are enrolled in short courses of
six months or less.
www.anujjindal.in

Key Findings of ASER 2023 ‘Beyond Basics’

Work outside home


• 33.7% of all youth aged 14-18 worked for more than 15 days in the last
month (excluding household work)

• Higher percentage of males (40.3%) than females (28%) are doing


work

• Of those who are in college/school, depending on age and grade,


between 29% to 37% work

• Of those who are not currently enrolled 55% work.


www.anujjindal.in

Key Findings of ASER 2023 ‘Beyond Basics’

Work at home

• More females (86%) reported working at home as compared to males


(66%)

• Among both males and females, most youth who are working in
activities other than household work tend to be working on family
farms.
www.anujjindal.in

Key Findings of ASER 2023 ‘Beyond Basics’


ABILITY

Foundational skills for youth in the age group of 14-18


• 73.6% of all youth in the age group 14 to 18 can read at least
Standard II level of text fluently in their regional language.
▪ 76% of females as compared to 71% of males can read at least
at Std II level.
▪ About 25% of this age group still cannot read a Std II level text
fluently in their regional language.

• Only 43.3% of all youth in the age group 14 to 18 can do division (3-
digit by 1-digit) problems. This skill is usually expected in Std III/IV.
▪ 45% of males can at least solve the division problem as
compared to 42% of females.
www.anujjindal.in

Key Findings of ASER 2023 ‘Beyond Basics’


• 57.3% of all youth can at least read English sentences.
▪ Of those who can read, 73.5% can understand the meaning of
the sentence
Everyday calculations
• Nearly 85% of surveyed youth in the age group of 14 to 18 years
can measure length using a scale when the starting point is 0 cm.
▪ This proportion drops sharply to 39% when the starting point is
moved.
▪ Overall, close to 50% youth can do other common calculations.
www.anujjindal.in

Key Findings of ASER 2023 ‘Beyond Basics’


Reading and understanding written instructions – daily life
applications
• Task: Youth were shown a picture of an O.R.S. packet and asked
some questions regarding the information given on it. This task was
administered only to those youth who could read at least a Std I
level text on the basic ASER reading assessment.
• Overall, 65% of youth could correctly answer at least 3 of the 4
questions.
▪ 69% of those who could read at least at Std II level answered
correctly as compared to 38.5% of those who were not able to
read at Std II level.
www.anujjindal.in

Key Findings of ASER 2023 ‘Beyond Basics’

Financial calculations
• Task: Youth who could do at least subtraction on
the ASER arithmetic test were asked to do some
commonplace financial calculations.
• Of the youth who can do subtraction or more,
over 60% can do the budget management task,
about 37% can apply a discount, but only about
10% can calculate repayment.
www.anujjindal.in

Key Findings of ASER 2023 ‘Beyond Basics’


Digital awareness and skill

It included self-reported questions capturing youths’ access to and


usage of digital devices, and an assessment of their digital skills –
actually doing a set of tasks in front of the survey team using an
available smartphone.
Availability/Access & ownership of Digital Devices (Self-reported)
• Computers
▪ Of all youth, 9% have a computer.
✓ Of those, 85% can use it.
▪ Of all youth, 91% do not have a computer.
✓ Of those 34% can use it.
www.anujjindal.in

Key Findings of ASER 2023 ‘Beyond Basics’


• Smartphone

▪ 89% of all youth have a smartphone at home


✓ 92% can use a smartphone
✓ Of those who can use a smartphone 31% have their own
smartphone.
✓ Of all males who can use a smartphone, 44% have their
own smartphone.
✓ Of all females who can use a smartphone, 20% have their
own smartphone.
www.anujjindal.in

Key Findings of ASER 2023 ‘Beyond Basics’

• Usage of Smartphones

▪ For Entertainment
✓ 78% used a smartphone for watching movies or listening to
songs
✓ 57% played games on a smartphone
✓ Visible gender differences: Males are more likely to have
accessed at least one of these services (37.6%) than
females (19%)
www.anujjindal.in

Key Findings of ASER 2023 ‘Beyond Basics’


▪ Communication and online safety (Self-reported)

✓ Almost all youth (90.5%) report having used social media in the
reference week.
➢ Slightly higher proportion of males (93.4%) than females
(87.8%) reported doing so.

✓ Of all youth who used social media, only about half are familiar
with the online safety settings that were included in the survey.
➢ Males are more likely to know about these settings than
females.
www.anujjindal.in

Key Findings of ASER 2023 ‘Beyond Basics’

▪ Education and Learning

✓ 61% of youth reported doing some education-related activity


online

✓ 45-50% watched videos related to studies, solved doubts using


online resources, and exchanged notes using messaging apps

✓ Higher usage by those who could at least read at Std II level


www.anujjindal.in

Key Findings of ASER 2023 ‘Beyond Basics’

▪ For Services
✓ 28% of youth reported ever using online services:
➢ 17% have made payments or filled out forms
➢ 7% have paid bills
➢ 4% have booked tickets
www.anujjindal.in

[a] 3 and 4 Only Q1: Which of the following findings about foundational skills for youth in the
[b] 1 and 5 Only age group of 14-18 are incorrect according to the recently released Annual Status
of Education Report (ASER) 2023 ‘Beyond Basics’?
[c] 1 and 2 Only
(1) 83.9% of all youth in this age group can read at least Standard II level of text
[d] 2 and 3 Only
fluently in their regional language
[e] 4 and 5 Only
(2) 76% of males in this age group can read at least Standard II level of text
fluently in their regional language as compared to 71% of females can read
(3) Only 43.3% of all youth in this age group can do division (3-digit by 1-digit)
problems
(4) 45% of males can at least solve the division problem as compared to 42% of
females
(5) Only 57.3% of youth in this age group can at least read English sentences

Solution: [c]
www.anujjindal.in

Q2: Identify incorrect about the recently released Annual Status of Education Report
(ASER) 2023 ‘Beyond Basics’
[a] It was released by the Ministry of Education
[b] It was conducted in 28 districts across 26 states, reaching a total of 34,745 youth in
the age group 14-18 years
[c] It looked ’beyond basics’ to explore a wider set of domains beyond foundational skills
[d] It explored domains such as Activity, Ability, and Awareness & Digital Aptitude
[e] None of the above

Solution: [a]
www.anujjindal.in

Q3: According to the Annual Status of Education Report (ASER) 2023 ‘Beyond Basics’
report, ___________ of 14-18-year-olds are enrolled in an educational institution. Only
about _______of all youth in this age group are doing vocational courses. Fill the gaps
[a] 69.3%, 6.4%
[b] 78.2%, 8.9%
[c] 80.8%, 7.7%
[d] 86.8%, 5.6%
[e] 92.4%, 10.5%

Solution: [d]
www.anujjindal.in

Q4: Identify the correct statement with respect to findings of the Annual Status of
Education Report (ASER) 2023 ‘Beyond Basics’
[a] Almost 80.5% of youth have used social media, a higher proportion amongst females
(93.4%) than males (87.8%)
[b] 61% of youth in the age group 14 to 18 have reported doing some education-related
activity online
[c] 45-50% watched videos related to studies, solved doubts using online resources, and
exchanged notes using messaging apps
[d] 28% of youth reported ever using online services
[e] 78% used a smartphone for watching movies or listening to songs

Solution: [a]
www.anujjindal.in

Q5: As per the availability/access & ownership of Digital Devices (Self-reported) findings
in the Annual Status of Education Report (ASER) 2023 ‘Beyond Basics’, of all youth in the
age group of 14-18 years, ________have a computer and ______ have a smartphone at
home. Fill the gaps
[a] 14%, 95%
[b] 9%, 89%
[c] 10%, 80%
[d] 8%, 75%
[e] 12%, 70%

Solution: [b]
PIB
24*7
25th– 27th January 2024
www.anujjindal.in

Q1: Which of the following statements are correct about


Subhash Chandra Bose Aapda Prabandhan Puraskar?
(1) It is an annual award instituted by the Ministry of Home
Affairs [a] 1 and 4 Only
(2) It was launched in 2022 on the 125th birth anniversary [b] 1, 2, and 4 Only
of Netaji Subhash Chandra Bose [c] 1, 3 and 4 Only
(3) It is given in only one category- Institution, carrying Rs. [d] 1, 2 and 3 Only
51 lakh cash prize and a certificate
[e] All of the above
(4) It recognizes and honors the invaluable contribution to
the field of disaster management

Solution: [b]
www.anujjindal.in

Subhash Chandra Bose


Aapda Prabandhan
Puraskar-2024
What: Ministry of Home Affairs has
selected 60 Parachute Field Hospital,
Uttar Pradesh, in the Institutional
category for the Subhash Chandra Bose
Aapda Prabandhan Puraskar-2024
Why: For its excellent work in Disaster
Management.
www.anujjindal.in

About 60 Parachute Field Hospital

• Mandate: To conduct Humanitarian Assistance and Disaster


Relief operations during natural calamities, nationally and
internationally, both in times of peace and war.

• Established Year: 1942

• It is the sole airborne medical establishment of the Indian


Armed Forces, recognized for its exceptional service in various
global crises.
www.anujjindal.in

About Subhash Chandra Bose Aapda


Prabandhan Puraskar
• Objective: To recognize and honour the invaluable contribution
and selfless service rendered by individuals and organizations in
India in the field of disaster management.

• Launch Year: In 2022 on the occasion of the 125th birth


anniversary of Netaji Subhash Chandra Bose.

• Nodal Ministry: Ministry of Home Affairs.


www.anujjindal.in

About Subhash Chandra Bose Aapda


Prabandhan Puraskar

• Categories: It is given in two categories- institution and


individual.

• It is an annual award announced on 23rd January, the birth


anniversary of Netaji Subhash Chandra Bose.

• Reward:
▪ Institution Category: Rs. 51 lakh cash prize and a certificate.
▪ Individual Category: Rs. 5 lakh prize and a certificate.
www.anujjindal.in

Q2: Ministry of Agriculture and Farmers Welfare in collaboration with which


organisation/s has recently launched the ‘Investment Forum for Advancing Climate
Resilient Agrifood Systems in India’?
[a] NITI Aayog
[b] World Bank
[c] UN-FAO
[d] A and B
[e] A and C

Solution: [e]
www.anujjindal.in

Investment Forum for


Advancing Climate Resilient
Agrifood Systems
What: NITI Aayog, the Ministry of
Agriculture and Farmers’ Welfare, and
the United Nations - Food and
Agriculture Organization (UN-FAO) have
jointly launched the ‘Investment Forum
for Advancing Climate Resilient Agrifood
Systems in India’ in New Delhi.
www.anujjindal.in

About Investment Forum

• Objective: To develop an investment and partnership strategy to


advance climate-resilient agri-food systems among the
government, private sectors, and farmers’ organizations and
financial institutions in India.

• It will facilitate the identification of national priorities and policy


platforms for financing climate-resilient agri-food systems
through a variety of investments.
www.anujjindal.in

About Investment Forum

• Six key focus areas

▪ Climate resilient agriculture (experiences and pathways)


▪ Digital infrastructure and solutions
▪ Financing climate resilient agrifood systems (domestic and
global)\Climate resilient value chains
▪ Production practices and inputs for climate resilience
▪ Gender mainstreaming and social inclusion for climate
resilience
www.anujjindal.in

Q3: How much outlay has been approved by the Union Cabinet for the Scheme for the
promotion of Coal/Lignite Gasification Projects of Government PSUs and the Private
Sector?
[a] Rs.7,000 crore
[b] Rs. 8,500 crore
[c] Rs. 9,500 crore
[d] Rs. 10,000 crore
[e] Rs. 11,000 crore

Solution: [b]
www.anujjindal.in

Scheme for promotion of


Coal/Lignite Gasification
Projects
What: The Union Cabinet has approved
the Scheme for the Promotion of
Coal/Lignite Gasification Projects of
Government PSUs and Private Sector.
www.anujjindal.in

About Scheme
• Total outlay: Rs.8,500 crores

▪ It will be provided as financial assistance for coal gasification


projects under three categories.
✓ Category I: Rs.4,050 crore provisioned for Government
PSUs in which upto 3 projects will be supported by
providing lump-sum grant of Rs.1,350 crore or 15% of
capex, whichever is lower.

✓ Category II: Rs.3,850 crore provisioned for private sector


as well as Government PSUs in which lump-sum grant of
Rs.1,000 crore or 15% of capex, whichever is lower
provided for each project.
www.anujjindal.in

About Scheme

✓ Category III: Rs.600 crore provisioned for demonstration


Projects (indigenous technology) and/or small-scale
product-based Gasification Plants under which lump-
sum grant of Rs.100 crore or 15% of capex, whichever is
lower.
www.anujjindal.in

Q4: Which Ministry has renewed its MoU with Lady Irwin College, University of Delhi to
work with the ROSHNI – Centre of Women Collectives-led Social Action to strengthen
interventions for better results of Food, Nutrition, Health, and WASH for rural
communities?
[a] Ministry of Women and Child Development
[b] Ministry of Health and Family Welfare
[c] Ministry of Rural Development
[d] Ministry of Panchayati Raj
[e] Ministry of Social Justice and Empowerment

Solution: [c]
www.anujjindal.in

MoU: DAY-NRLM & Lady


Irwin College
What: DAY-NRLM and Lady Irwin College,
University of Delhi, have renewed a
Memorandum of Understanding (MoU) for
5-year term.
Why: To work with ROSHNI – Centre of
Women Collectives led Social Action
(CWCSA) to strengthen interventions for
better results of Food, Nutrition, Health,
and WASH for rural communities.
www.anujjindal.in

About ROSHNI-CWCSA

• It is technically and financially supported by UNICEF India.

• It serves as a technical support unit at the national level to DAY-


NRLM and the Department of Development Communication and
Extension, Lady Irwin College.

• It assists DAY-NRLM to improve food, nutrition, health, and


WASH outcomes for 9.96 crore Self Help Group (SHG) members
and their households across the country.
www.anujjindal.in

Q5: According to the Direct Tax Statistics (updated up to F.Y. 2022-23) released by the
Central Board of Direct Taxes, the Direct Tax to GDP ratio has increased from 5.62% in F.Y.
2013-14 to ________ in F.Y. 2022-23. Fill the gap
[a] 6.11%
[b] 7.35%
[c] 8.44%
[d] 9.56%
[e] 10.23%

Solution: [a]
www.anujjindal.in

Direct Tax Statistics


through Time-Series Data
What: The Central Board of Direct Taxes
(CBDT) has released Key Direct Tax
Statistics as updated up to F.Y. 2022-23.
www.anujjindal.in

Key highlights of Statistics


• Net Direct Tax Collections: Rs. 16,63,686 crores in F.Y. 2022-23
▪ It has increased by 160.52% from Rs. 6,38,596 crores in F.Y.
2013-14

• Gross Direct Tax Collections: Rs. 19,72,248 crores in F.Y. 2022-23


▪ They have registered an increase of over 173.31% compared
to Gross Direct Tax Collections of Rs. 7,21,604 crores in F.Y.
2013-14.

• Direct Tax to GDP ratio: It has increased from 5.62% in F.Y. 2013-
14 to 6.11% in F.Y. 2022-23.
www.anujjindal.in

Key highlights of Statistics

• Cost of collection: It has decreased from 0.57% of total


collection in the F.Y. 2013-14 to 0.51% of total collection in the
F.Y. 2022-23.

• Total number of ITRs filed: In FY 2022-23, it stands at 7.78 crore


showing an increase of 104.91% as compared to the total
number of ITRs of 3.80 crore filed in FY 2013-14.
www.anujjindal.in
www.anujjindal.in

Q6: Which organization has signed MoUs with four Sector Skill Councils to strengthen
the skill ecosystem in the country?
[a] ISRO
[b] DRDO
[c] CSIR
[d] ICAR
[e] NABARD

Solution: [c]
www.anujjindal.in

Q7: What is the name of the unified legal interface for legal information, legal advice,
and legal assistance launched recently by the Ministry of Law and Justice to extend and
expand the reach of legal services up to the last mile?

[a] Nyaya Setu


[b] Nyaya Drishti
[c] Nyaya Samridhi
[d] Nyaya Suvidha
[e] Nyaya Samarthan

Solution: [a]
www.anujjindal.in

Q8: Which Ministry has joined hands with YuWaah - UNICEF to empower young people
in urban governance through civic participation?
[a] Ministry of Education
[b] Ministry of Women and Child Development
[c] Ministry of Skill Development and Entrepreneurship
[d] Ministry of Defence
[e] Ministry of Housing and Urban Affairs

Solution: [e]
www.anujjindal.in

Q9: What is the total outlay of the PLI Scheme on 'National Programme on Advanced
Chemistry Cell (ACC) Battery Storage’ for achieving a manufacturing capacity of 50
gigawatt hours (GWh) of ACC launched in 2021?
[a] Rs. 10,855 Crore
[b] Rs. 12,000 Crore
[c] Rs. 14,100 Crore
[d] Rs. 16,300 Crore
[e] Rs.18,100 Crore

Solution: [e]
www.anujjindal.in

Q10: The Union Cabinet has appraised the MoU on cooperation in the field of
Information Technology between India and which country?
[a] Oman
[b] Saudi Arabia
[c] UAE
[d] Qatar
[e] Kuwait

Solution: [a]
PIB
24*7
28th– 29th January 2024
www.anujjindal.in

PM YASASVI Scheme for


OBC, EBC, and DNT
Students
What: PM Young Achievers’ Scholarship
Award Scheme for a Vibrant India (PM
YASASVI) is the umbrella Scholarship
Scheme for Other Backward Class
(OBCs), Economically Backward Class
(EBC) and Denotified Nomadic Tribes
(DNT) Students.
www.anujjindal.in

About PM YASASVI Scheme


• It is an umbrella Scheme formulated for OBC, EBC, and DNT
Students by clubbing the existing Scholarship Schemes and
Hostel Scheme
• Nodal Ministry: Ministry of Social Justice and Empowerment
• Duration: 2021-22 to 2025-26
• Five Subcomponents
▪ Pre-Matric Scholarship for OBC, EBC and DNT Students
▪ Post-Matric Scholarship for OBC, EBC and DNT Students
▪ Top Class School Education for OBC, EBC and DNT Students
▪ Top Class College Education for OBC, EBC and DNT Students
▪ Construction of Hostel for OBC Boys and Girls
www.anujjindal.in

Pre-Matric Scholarship for OBC, EBC and DNT Students


▪ Objective: To provide welfare and motivation to the children
from OBC, EBC, and DNT categories by providing financial
assistance at pre-matriculation or secondary stage
▪ Type: Centrally Sponsored Component/Scheme
▪ Eligibility Criteria for Students
✓ Income of Parents/Guardians: Less than Rs.2,50,000/- per
annum
✓ Students studying in classes IX and X on a full-time basis in
Government Schools only.
✓ Scholarship holder under it must not hold any other
educational scholarship for studying in class IX and X
✓ Scholarship for studying in any class will be available for only
one year
▪ Academic Allowance: Rs. 4000/- per annum.
www.anujjindal.in

Post-Matric Scholarship for OBC, EBC and DNT Students


Other conditions for Scholarship

• If a student violates school discipline or any other terms and


conditions of the scholarship, scholarship may be suspended or
even cancelled.
• Migration of students from one institution to another would not
normally be allowed during the course of academic year.
• Only two boys of the same parents/guardian will be entitled to
receive scholarships. This restriction will, however, not apply to
girls
• At least 5% students with disabilities should be covered for grant
of the scholarships.
• 30% of the beneficiaries should be reserved for girl students
www.anujjindal.in

Post-Matric Scholarship for OBC, EBC and DNT Students

▪ Objective: To provide financial assistance to the OBC, EBC, and


DNT students studying at the post-matriculation or post-
secondary stage to enable them to complete their education.

▪ Under it, a scholarship is awarded to students studying at post-


matriculation or post-secondary stage in recognized institutions
to enable them to complete their education.
www.anujjindal.in

Post-Matric Scholarship for OBC, EBC and DNT Students


▪ Eligibility Criteria for Students

• The scholarships will be open to Indian nationals.


• Students pursuing Post Graduation courses in medicine will be
eligible only if they are not allowed to practice during the
period of their course
• Only two boys of the same parents/guardian will be entitled to
receive scholarships. This restriction will, however, not apply to
girls
• A scholarship holder under this Scheme will not hold any other
scholarship/ stipend.
• Employed/Unemployed – Income should not exceed Rs.2.5
Lakhs per annum
www.anujjindal.in

Post-Matric Scholarship for OBC, EBC and DNT Students


• Academic Allowance: It includes the following components for
the complete duration of the course
www.anujjindal.in

Top Class School Education for OBC, EBC and DNT


Students
▪ Objective: To provide premium education to the meritorious
students belonging to OBC, EBC, and DNT categories by funding
their education from Class 9 onwards till they complete Class 12.

▪ Eligibility Criteria for Students


✓ Income of Parents/Guardians: Less than Rs.2,50,000/- per
annum
✓ 30% scholarship reserved for girl students

▪ Scholarship Benefits: Scholarship is awarded for tuition fees,


hostel fees, and other charges as required by the school.
✓ Rs. 75,000/- per annum per student of classes 9 and 10
✓ Rs. 1,25,000/- per annum per student of classes 11 and 12
www.anujjindal.in

Top Class Education in College For OBC, EBC and DNT


Students
▪ Objective: To recognize and promote quality education amongst
Students belonging to OBC, EBC, and DNT categories by
providing full financial support.
▪ It will cover OBC/EBC/DNT students for pursuing studies
beyond class XII

▪ Eligibility Criteria for Students


✓ Income from all sources should not exceed Rs.2,50,000/- per
annum.
✓ 30% of slots allotted to the Institution shall be reserved for
eligible girl
✓ The benefit of the Scheme will not be provided to more than
2 siblings in a family
www.anujjindal.in

Top Class Education in College For OBC, EBC and DNT


Students

Component Amount of assistance


Tuition fee and non-refundable • For private sector institutions –
charges Rs.2 Lakh (Max)
• For Commercial Pilot courses –
Rs.3.75 Lakh (Max)

Living expenses Rs.3000 per month per student


Books and Stationery Rs.5000 per annum per student
Laptop/computer Rs.45000 maximum
www.anujjindal.in

Construction of Hostels for OBC Boys and Girls


▪ Objective: To provide hostel facilities to students belonging to
socially and educationally backward classes, especially from
rural areas, to enable them to pursue secondary and higher
education.

▪ Cost Norms: The cost per hostel seat in different areas are as
follows
✓ North Eastern Region/Himalayan Regions: Rs.3.50 lakh per
seat
✓ Rest of country - Rs.3.00 lakh per seat
✓ The amount of grant will be released in 3 instalments in
50:45:5 ratio
www.anujjindal.in

Construction of Hostels for OBC Boys and Girls

▪ Cost Sharing Pattern:

• Centre: States – 60:40 – For construction of boys hostel


• Centre: States – 90:10 – For construction of girls hostel
• Centre: N-E states and Himalayan States – 90:10
• Centre: UT – 100:00
• In case of reputed institute, ranked in the latest NIRF
rankings, the Central Assistance shall be 100% irrespective
of regional States
www.anujjindal.in

Q2: Which initiative will be launched by the Ministry of Electronics & Information
Technology to enable partnerships between the Government, startups and large
enterprises in the electronics space?
[a] Digital India futureLABS
[b] Digital India eLABS
[c] Digital India eConnect
[d] Digital India mConnect
[e] Digital India eFacilitation

Solution: [a]
www.anujjindal.in

Digital India futureLABS

What: The Ministry of Electronics &


Information Technology will launch
Digital India futureLABS
Why: To enable partnerships between
the Government, startups and large
enterprises in the electronics space
www.anujjindal.in

About Digital India futureLABS


• Objective: To boost India’s Electronics & IT sector by establishing
a research and innovation framework, promoting leadership in
standards, IPs, systems, and platforms.

• Nodal Agency: C-DAC

• It will focus on strengthening the domestic innovation


ecosystem through collaboration, driving sustainable growth,
and fostering technological progress.

• It will concentrate on sectors like Automotive, Mobility,


Communication, Strategic Electronics, and Industrial IoT.
www.anujjindal.in
www.anujjindal.in

Q3: The Union Cabinet has approved the protocol for the establishment of the Joint
Economic and Trade Committee (JETCO) between India and which country?
[a] Dominican Republic
[b] Iran
[c] Saudi Arabia
[d] France
[e] Canada

Solution: [a]
www.anujjindal.in

Q4: Which PSU has recently signed a MoU with the National Investment and
Infrastructure Fund Limited (NIIFL) to collaborate on a suite of funding solutions for
Renewable Energy Projects as well as large-scale infrastructure projects in India?
[a] BPCL
[b] BHEL
[c] REC Limited
[d] ONGC Limited
[e] BPCL

Solution: [c]
www.anujjindal.in

Q5: Which partner agency/ies of the Central Board of Indirect Taxes and Customs
(CBIC) was/were awarded on the occasion of International Customs Day for their
partnership and association with the Customs department in improvement in cross-
border cargo clearance time?
[a] Food Safety and Standards Authority of India
[b] Central Drugs Standard Control Organisation
[c] Department for Promotion of Industry and Internal Trade
[d] A and B
[e] A, B and C

Solution: [d]
www.anujjindal.in

Q6: What is the name of the indigenously developed India’s first women-friendly
compact electric tractor of CSIR?
[a] ECROP R89
[b] EFARMA E67
[c] TETRA EO45
[d] PRIMA ET11
[e] ELECTRA EM34

Solution: [d]
www.anujjindal.in

Q7: The Ministry of Education has recently launched the registration of Phase IV of
Yuva Sangam, under Ek Bharat Shreshtha Bharat (EBSB). Yuva Sangam is an initiative
by the Government of India to strengthen people-to-people connections between
youth in the age group of ____________, belonging to different States/UTs of India.
Fill the gaps
[a] 13-25 years
[b] 15-28 years
[c] 18-30 years
[d] 10-25 years
[e] 10-30 years

Solution: [c]
www.anujjindal.in

Q8: Which Ministry’s Tableau has won the first prize at the Republic Day Parade
2024?
[a] Ministry of Defence
[b] Ministry of Culture
[c] Ministry of Electronics and Information Technology
[d] Ministry of Education
[e] Ministry of Women and Child Development

Solution: [b]
www.anujjindal.in

Q9: Which of the following is India’s nomination for recognition as UNESCO World
Heritage List for the year 2024-25?
[a] Maratha Military Landscapes of India
[b] Temples at Bishnupur
[c] Mattanchery Palace
[d] River Island of Majuli in midstream of Brahmaputra River
[e] Sri Harimandir Sahib

Solution: [a]
www.anujjindal.in

Q10: Which company has been certified as the 2024 Top Employers in India by the
Top Employers Institute?
[a] ONGC
[b] REC
[c] BPCL
[d] BHEL
[e] NTPC

Solution: [e]
PIB
24*7
30th– 31st January 2024
www.anujjindal.in

Q1: Which Ministry/ies recently organized the National Conference on Strategic Trade
Controls (NCSTC) in New Delhi with a focus on India’s Strategic Trade Control related
to Special Chemicals, Organisms, Materials, Equipment, and Technologies (SCOMET)
and Export Controls system?
[a] Ministry of Commerce and Industry
[b] Ministry of Finance
[c] Ministry of External Affairs
[d] A and B
[e] A and C

Solution: [e]
www.anujjindal.in

National Conference on
Strategic Trade Controls
The Directorate General of Foreign
Trade, Ministry of Commerce & Industry
in partnership with the Ministry of
External Affairs has organized the
National Conference on Strategic Trade
Controls in New Delhi.
www.anujjindal.in

About Conference

• It focussed on India's Strategic Trade Control related to Special


Chemicals, Organisms, Materials, Equipment and Technologies
(SCOMET) and Export Controls system.

• It also emphasized on ensuring compliance related to the export


of dual-use (industrial and military) goods, software, and
technologies.
www.anujjindal.in

Background
• Strategic Trade Control: It deals with items that are subject to
licensing or authorization such as chemical and biological
materials.
• Dual-use items: These are goods and technologies that can have
both civilian and military applications.
▪ These items have been named as SCOMET under Foreign
Trade Policy, 2023.
• Export of dual-use items, including software and technologies, is
either prohibited or is permitted under an Authorization unless
specifically exempted.
• SCOMET List is notified under Indian Trade Clarification based on
Harmonized System (ITC HS) Classification.
www.anujjindal.in

Q2: Which integrated portal has been launched by the Ministry of Communications to
propel India's 5G capabilities, and foster innovation, collaboration, and knowledge-
sharing within the telecom sector?
[a] 5G for Growth Portal
[b] 5G for India Portal
[c] Bharat 5G Portal
[d] 5G Digital Growth Portal
[e] Digital 5G India Portal

Solution: [c]
www.anujjindal.in

Bharat 5G Portal
The Ministry of Communications has
launched the “Bharat 5G Portal- an
integrated portal” on the sidelines of
‘Bharat Telecom 2024 - An Exclusive
International Business Expo’ held in
New Delhi
www.anujjindal.in

About Bharat 5G Portal


• Objective: To propel India's 5G capabilities, and foster
innovation, collaboration, and knowledge-sharing within the
telecom sector.

• Developed by: Telecommunications Consultants India Limited


▪ It is a central public sector undertaking under the ownership
of the Department of Telecommunications, Ministry of
Communications

• It is a comprehensive platform to serve the interests of startups,


industry, and academia in quantum, 6G, Intellectual Property
Rights, and 5G domains.
www.anujjindal.in

Q3:What is the approved cost of recently launched Computerization Scheme for the
offices of the Registrar of Cooperative Societies (RCSs) and Agriculture and Rural
Development Banks (ARDBs) of the states?
[a] Rs. 123 crores
[b] Rs. 225 crores
[c] Rs. 321 crores
[d] Rs. 465 crores
[e] Rs. 523 crores

Solution: [b]
www.anujjindal.in

Computerization Scheme
for the offices of RCSs and
ARDBs
The Ministry of Cooperation has
launched the Computerization Scheme
for the offices of the Registrar of
Cooperative Societies (RCSs) and
Agriculture and Rural Development
Banks (ARDBs) of the states.
www.anujjindal.in

About Computerization Scheme


• Objective: To standardize business processes, enhance
operational efficiency, and bring transparency to ARDB
operations.

• Computerization project of ARDBs: It seeks to link 1851 units,


spanning 13 States/UTs with NABARD through Common
National Software.

• Cost: Rs.225 crore, out of which Rs.120 crore will be spent on


ARDBs and Rs.95 crore will be spent on RCSs
www.anujjindal.in

Q4: Which initiative has been launched recently by the Ministry of Education and
AICTE to foster innovation and entrepreneurship in the education sector?
[a] Investment Fund for Education
[b] Investment Network for Education
[c] Education Investment Fund
[d] Pragyaan Innovation Centre
[e] Investor Network

Solution: [e]
www.anujjindal.in

MoE - AICTE Investor


Network
What: The Ministry of Education - AICTE
Investor Network has been launched
recently.
Why: To create a vibrant ecosystem for
innovation and entrepreneurship in
education sector.
www.anujjindal.in

About MoE - AICTE Investor Network

• Objective: To provide crucial financial support, mentoring, and


strategic guidance to early-stage student or faculty-led startups.

• It has been jointly established by AICTE and the Ministry of


Education’s Innovation Cell.

• It will bridge the gap between investors and transformative


educational initiatives.
www.anujjindal.in

Q5: Which is the following is the implementing agency of the National Mission for
Mentoring launched in 2022 to create a large pool of outstanding professionals
willing to provide professional and personal support to school teachers across the
country?
[a] AICTE
[b] UGC
[c] KVS
[d] NCTE
[e] NCERT

Solution: [d]
www.anujjindal.in

National Mission for


Mentoring
What: The National Council for Teacher
Education (NCTE) has organized a
seminar on the National Mission for
Mentoring (NMM)
www.anujjindal.in

About National Mission for Mentoring


• Objective: To create a network of mentors who will guide and
support teachers in developing the necessary 21st-century skills.

• Origin: It has been envisaged in para 15.11 of the National


Education Policy (NEP 2020)

• Launch Year: 2022

• Implementing Agency: National Council for Teacher Education

• It has been established to create a large pool of outstanding


professionals willing to provide professional and personal
support to school teachers across the country and ensure their
continuous professional development.
www.anujjindal.in
www.anujjindal.in

Q6: Which state is hosting the 2nd Edition of India Energy Week in February 2024?
[a] Goa
[b] Tamil Nadu
[c] Maharashtra
[d] Karnataka
[e] Gujarat

Solution: [a]
www.anujjindal.in

Q7: When was the National AYUSH Morbidity and Standardized Terminologies
Electronic Portal (NAMASTE Portal) launched by the Ministry of AYUSH to provide
standardized terminologies and morbidity codes for Ayurveda, Siddha, and Unani
systems of medicines?
[a] 2015
[b] 2017
[c] 2019
[d] 2021
[e] 2023

Solution: [b]
www.anujjindal.in

Q8: Ministry of Commerce and Industry in collaboration with which industry body
recently organized the 'PM GatiShakti Summit: Towards Logistics Efficiency and
Integrated Infrastructure Planning' in New Delhi?
[a] CII
[b] ASSOCHAM
[c] FICCI
[d] CII
[e] FIEO

Solution: [c]
www.anujjindal.in

Q9: The gross GST revenue collected in January 2024 at Rs.1,72,129 crore was the
second-highest monthly collection ever. When the highest-ever GST collection was
achieved?
[a] December 2023
[b] October 2023
[c] April 2023
[d] June 2023
[e] August 2023

Solution: [c]
www.anujjindal.in

Q10: Which 3-month-long accelerator program has been launched by DPIIT under
Startup India for early-stage startups to provide them access to knowledge, network,
funds, and guidance required to scale up?
[a] StartupShala
[b] StartupEdu
[c] StartupInfo
[d] StartupShiksha
[e] StartupSamridhi

Solution: [a]

You might also like